Hematology NEJM
Hematology NEJM
A 28-year-old woman is evaluated for new thrombocytopenia. She was Which one of the fo llowing medications or conditions is the most likely
hospitalized 20 days ago w ith pyelonephritis and received intravenous cause of the patient's low platelet count?
piperacillin-tazobactam for the pyelonephritis and enoxaparin for deep-vein
thrombosis prophylaxis. She was discharged 11 days ago, at which time she Not there yet...
started a course of trimethoprim-sulfamethoxazole. Her only other
medication is omeprazole, which she has taken for several years.
64% Correct Trimethoprim-sulfamethoxazole
Her vital signs are within normal limits. Her only current symptom is easy
bruising. She does not have any leg swelling or other symptoms of 22% Wrong Enoxaparin
thrombosis.
Her platelet count was 155,000 per mm 3 (reference range, 150,000-350,000) 2% Other Sepsis
at hospital discharge and is now 14,000 per mm 3 . Other current laboratory
results are as follows: 2% Other Omeprazole
DETAILED FEEDBACK:
CLOSE
Copyrigh t © 2022 Massachusetts Medical Society. All rights reserved
(D You are in Demo mode. Results are not saved.
A 24-year-old woman is diagnosed with a pulmonary embolism after Which one of the following tests should be used to monitor t herapy with
hospitalization for open reduction and internal fixation of a fractured tibia. She unfractionated heparin in this patient?
has a history of lupus anticoagu lant.
Laboratory testing performed before the admin istration of unfractionated Not t here yet...
heparin reveals a baseline international normalized ratio of 1.1 and the
fol lowing other results: 39% Correct Anti-factor Xa assay
Patient value Reference range
3% Wrong Bleeding time
Leukocyte count (per mm3) 13,600 4500-11,000
Fibrinogen (mg/dL) 450 150-400 The most appropriate test for monitoring unfractionated heparin therapy in
a patient with a prolonged baseline activated partial-thromboplastin time is
anti-factor Xa assay.
DETAILED FEEDBACK:
CLOSE
Copyright © 2022 Massachusetts Medical Society. All rights reserved
(D You are in Demo mode. Results are not saved.
A 65-year-old man with a history of atrial fibrillation, chronic kidney disease, Which one of the following management approaches is most appropriate
and congestive heart failure is hospitalized for an exacerbation of congestive regarding this patient's supratherapeutic international normalized ratio?
heart failure. He is maintained on warfarin with a target international
normalized ratio of 2.0 to 3.0. His liver is palpable 2 cm below the right costal Not there yet.
margin. There is 2+ pitting pretibial and ankle edema bilaterally.
His current international normalized ratio is 11. There is no evidence of Hold the warfarin and administer 2.5 mg of vitamin K
68% Correct
bleeding. by mouth
DETAILED FEEDBACK:
CLOSE
Copyright © 2022 Massachusetts Medical Society. All rights reserved
(D You are in Demo mode. Results are not saved.
A 51-year-old woman is hospitalized for an elect ive left knee replacement. One
day after surgery, her hospital course is complicated by a large ecchymosis at
the incision site, severe swelling of the left knee with excruciat ing pain, and
continued postoperative bleeding from the wound.
Her hemoglobin level is 10.0 g/dl (reference range, 12.0-16.0), compared 53% Right I Administer recombinant human factor VIia
with a preoperative value of 14.2 g/dl.
Other postoperat ive laboratory results are as follows: 2% Other Perform a platelet transfusion
Partial-thromboplastin time, act ivated (sec) 45.0 22. 1-35. 1 20% Other Administer fresh frozen plasma
Prothrombin time (sec) 120 11.1-13. 1
Mixing the patient's plasma 1:1 with plasma that contains 100% of the normal A prolonged activated partial-thromboplastin t ime that is not corrected by a
factor level does not change the activated partial-thromboplastin t ime. A mixing study suggests the presence of a factor inhibitor, most commonly a
platelet-function screen is normal. factor VIII inhibitor.
DETAILED FEEDBACK:
The fact that the presence of adequate levels of clotting factors does not
correct the activated partial-thromboplastin time in the mixing study
suggests that the patient's plasma contains a factor that inhibits one of the
clotting factors, most commonly factor VIII. Most acquired inhibitors are
antibodies. Treatment of acquired inhibitors of coagulation factors should
have two goals: controlling the bleeding and eliminating the inhibitor. Given
that this patient is bleeding excessively after surgery, administration of
either activated prothrombin complex concentrates or recom binant human
factor VIia is required in order to bypass t he factor VIII inhibitor.
In the long term, the patient w ill require prednisone (and/ or additional
immunosuppressive therapy) to control the autoimmune inhibitor, but that
would take too long to control the acute bleeding.
Given that the patient's platelet count and function are normal, a platelet
tr· r c:f IsIon VVOL Id not hPlp r.ontr01 hP blPi>ci1nq
CLOSE
Copyright © 2022 Massachusetts Medical Society_All rights reserved
(D You are in Demo mode. Results are not saved.
What is the most appropriate strategy for assessing the bleeding risk in a 25-year-old patient who is scheduled for elective inguinal hernia repair and reports an
occasional history of easy bruising on the extremities?
76% Correct Obtain a thorough personal and family medical history and bleeding history
The most important way to assess bleeding risk before elect ive surgery is to obtain a thorough medical history and bleeding history.
DETAILED FEEDBACK:
In assessing bleeding risk, a careful medical and surgical history is of primary importance. In the absence of a personal or family bleeding history (including
features that might suggest diagnoses such as Marfan syndrome, Ehlers-Danlos syndrome, or Cushing syndrome), preoperative coagulation testing is not
indicated for surgeries with low bleeding risk. If the personal history or physical examination suggests an underlying bleeding problem, or if the procedure is high-
risk (e.g., neurosurgery), t hen preoperative testing is appropriate. A patient's report of "easy bruising" without a history of more substantial or serious bleeding is
not generally an indication for addit ional evaluat ion.
If the patient gives a strong fam ily history of a bleeding disorder, the c linician should obtain more information before deciding which addit ional tests (if any) to
order.
Preoperative coagulation studies are reserved for patients who have a history of bleeding or medical conditions that predispose them to bleeding and for those
taking anticoagulants.
Bleeding t ime is a test of platelet function with limited clinical utility and is not offered in most clinical laboratories.
A plat elet-function assay measures both platelet adhesion and aggregat ion. It is often used to screen for von Willebrand disease or other inherited platelet function
defects.
Last reviewed Feb 2021.
CITATIONS:
Feely MA et al. Preoperative testing before noncardiac surgery: guidelines and recommendat ions. Am Fam Physician 2013 Apr 4; 87:414. PMID: 23547574
CLOSE
Copynght © 2022 Massachusetts Medical Society. All nghts reserved
(D You are in Demo mode. Results are not saved.
A 44-year-old man of Irish heritage was evaluated 6 months ago for type 2 Which one of t he following next steps is most appropriate in th is patient's
diabetes, hypertension, elevated aminotransferase levels, erectile dysfunction, care?
and soft, small testes. Mutation testing of the HFE gene confirmed the
presence of a homozygous C282Y mutat ion, and he was initiated on a Not t here yet...
schedule of weekly therapeutic phlebotomy with removal of 500 ml of blood.
His most recent laboratory results show a hemoglobin level of 11.7 g/dl Continue therapeutic phlebotomy at a reduced
1% Correct
(reference range, 13.5-17.5), a ferritin level of 433 ng/ml (30-300), and a frequency
transferrin saturat ion of 43% (15-45). He is currently asymptomatic.
Discontinue phlebotomy, and restrict dietary iron
33% Wrong
intake
DETAILED FEEDBACK:
CLOSE
Copyright © 2022 Massachusetts Medical Society_ All rights reserved
G) You are in Demo mode. Results are not saved.
A 29-year-old man is admitted to the hospital for right tibial osteotomy after Which one of the following approaches to venous thromboembolism
he develops complications related to a right-knee meniscectomy. He is prophylaxis is most appropriate for this patient?
expected to be hospitalized for 2 to 3 days and unable to ambulate for 2 to 3
weeks. Wou got 1tI
45% Right
I Low-dose daily rivaroxaban starting 12 hours after
the procedure and continued for 3 weeks
For young patients at low bleeding risk who are undergoing major
orthopedic surgery, the appropriate means of venous thromboembolism
prophylaxis is a daily low-dose direct oral anticoagulant beginning 12 hours
after surgery and continued until the patient regains ambulatory function.
DETAILED FEEDBACK:
CLOSE
Copynght © 2022 Massachusetts Medical Society. All nghts reserved
(D You are in Demo mode. Results are not saved.
A 47-year-old man is evaluated in the emergency department for severe Which one of the following diagnoses is the most likely cause of this
epistaxis and oral mucosal bleeding. He has developmental delay but no patient's coagulopathy?
significant medical history and is taking no medications except for a daily
multivitamin. Not there yet...
His international normalized ratio (INR) is 10.2, and his partial-thromboplastin
time is >300 seconds (reference range, 22.1-35.1 ). His liver function test 66% Correct Superwarfarin (rat poison) intoxication
results are normal. He is given 4 units of fresh frozen plasma and 10 mg of
vitamin K liquid by mouth. 4% Wrong Hemophilia A
The following day, his INR is 1.9. However, 2 days later while still in the
hospital, he develops gross hematuria and his INR is 4.8. At that time, 14% Other Acquired von Willebrand syndrome
additional laboratory evaluation shows decreased levels of factors II, VII, IX,
and X. His D-dimer, thrombin t ime, and levels of factors V. VIII, XI, and XI I are 5% Other Hemophilia B
normal. Mixing studies are negative fo r a prothrombin time inhibitor, a partial-
thromboplastin time inhibitor, and lupus anticoagulant 10% Other Warfarin overdose
He is managed with further fresh frozen plasma transfusions and daily
vitamin K. His coagulopathy improves over several months with ongoing
KEY LEARNING POINT:
tapering of vitamin K therapy.
The cause of a prolonged, reversible coagulopathy characterized by a high
international normalized ratio and deficiencies of factors II, VII, IX, and X
should raise suspicion for superwarfarin (rat poison) intoxication.
DETAILED FEEDBACK:
CLOSE
Copynght © 2022 Massachusetts Medical Society_ All nghts reserved
G) You are in Demo mode. Results are not saved.
A 45-year-old man reports a 3-day history of left lower-extremity swelling and Which one of the following management approaches is best for this
pain and is found to have a common femoral deep-vein thrombosis. The patient?
patient could not identify a precipitating event and has otherwise been in good
health. He takes no medications. Not there yet..
Laboratory evaluation documents a normal prothrombin time and an
activated partial- thromboplastin time {a PTT) of 80.0 sec (reference range, Initiate low-molecular-weight heparin, then transition to
22.1-35.1). 83% Correct indefi nite warfarin therapy with an international
When the patient's plasma is mixed with normal plasma in a 1:1 ratio, the norma lized ratio goal of 2.0 to 3.0
aPTT fails to correct. When excess phospholipid is added to the patient's
plasma, the aPTT corrects. Initiate low-molecular-weight heparin, then transition
3% Wrong to 6 months of warfarin therapy with an international
normalized ratio goal of 3 .0 to 4 .0
DETAILED FEEDBACK:
CLOSE
Copyright © 2022 Massachusetts Medical Society. All nghts reserved
G) You are in Demo mode. Results are not saved.
A 25-year-old woman reports occasional nosebleeds since childhood t hat What Is the most likely d1agnos1s In this case?
have worsened recently. She says that her menstrual periods have always
been heavy but exceptionally so during the past 2 months. Not there yet. II
She has no other history of medical problems. She takes aspirin occasionally
for headaches and uses oral contraceptive pi lls. She was adopted as a child, 69% Correct Factor VIII deficiency
and no family history is available.
A complete blood count is within normal limits. The prothrombin t ime is 12.0 7% Wrong Lupus anticoagulant
seconds (reference range, 11.1-13.1 ), and the activated partial-
thromboplastin time is 48.0 seconds (221-35.1 ). 12% Other Factor XI I deficiency
DETAILED FEEDBACK:
CLOSE
Copyright © 2022 Massachusetts Medical Society. All nghts reserved
(D You are in Demo mode. Results are not saved.
A 35-year-old woman is scheduled for cholecystectomy after presenting one Which one of the following next steps is most appropriate in the care of this
month ago with typical symptoms of cholecystitis. She has a history of an patient?
uncom plicated tooth extract ion and appendectomy in the past. She has no
family history of bleeding and denies spontaneous bleeding. Not there yet...
Her international normalized ratio is 1 0 Ot her preoperative laboratory results
are as follows: 67% Correct I Proceed with surgery without additional treatment
Patient value Reference range
2% Wrong
I Proceed with surgery, but administer vitamin K before
Leukocyte count (per mm3) 8500 4500-1 1,000 the procedure
Hemoglobin (g/dL) 129 12.0-16.0
1% Other
I Proceed with surgery, but transfuse platelets before
Platelet count (per mm 3) 145,000 150,000-350,000
the procedure
Prothrombin t ime (sec) 120 11.1-13.1
Proceed with surgery, but give fresh frozen plasma
Partial-thromboplastin time (sec) 110.0 22.1-35.1 17% Other
before the procedure
A partial-thromboplastin time mixing study is consistent wit h a factor
Proceed with surgery, but administer cryoprecipitate
deficiency. Assays of specific factors reveal a factor XI I level of 15% (reference 13% Other
range, 60-140). before the procedure
DETAILED FEEDBACK:
CLOSE
Copynght © 2022 Massachusetts Medical Society. All nghts reserved
G) You are in Demo mode. Results are not saved.
A 25-year-old woman who is 3 weeks postpartum presents w ith a severe Which one of the following conditions is the most likely cause of this
headache. She has no history of trauma and takes no medications. On further patient's bleeding?
quest ioning, she notes that she had no problems with her recent delivery by
caesarean section but has noticed increased gum bleeding with floss ing Not there yet..
during the past week. She denies any previous bleeding or family history of
bleeding
56% Correct Acquired factor VIII inhibitor
A complete blood count is within normal limits. Her prothrombin t ime is
normal, but the activated partial-thromboplastin t ime is 90 seconds (reference 5% Wrong Factor XIII deficiency
range, 22.1-35.1 ). Non contrast CT of t he head demonstrates a small ri ght
subdural hematoma. 9% Other Lupus anticoagulant disorder
DETAILED FEEDBACK:
The pat ient has an acquired bleeding disorder; acquired factor VI II inhibitor
is the most likely et iology. Autoantibodies to factor VII I may be seen in
association with autoimmune syndromes or other diseases. One of the
more frequent presentations is in t he postpartum period, generally
attributed to pregnancy-related changes in the immune system.
Lupus anticoagulant disorder and factor XII deficiency are associated with
prolonged activated partial-thromboplastin time, but they are not
associated with increased bleeding risk.
Type I von Willebrand disease is an inherited disorder, so t he patient would
be likely to have a history of increased bleeding and a family history of
bleeding.
f
CLOSE
Copyright © 2022 Massachusetts Medical Society. All nghts reserved
(D You are in Demo mode. Results are not saved.
A 20-year-old female college student reports having heavy menstrual bleeding What d1agnos1s Is most likely In this case?
since puberty. She has previously developed several large hematomas after
minor injuries. She recalls an episode of extensive bleeding when her wisdom Not there yet.. II
teeth were removed. She has never had any other surgical procedures and
says she does not use aspirin or other nonsteroidal antiinflammatory drugs.
60% Correct von Willebrand disease
Her platelet function is normal, and her internat ional normalized ratio is 0.9.
Other laboratory results are as follows 16% Wrong Hemophilia B
Factor IX activity(%) 97 60-140 The most likely diagnosis in a young adult with recurrent bleeding whose
partial-thromboplastin t ime mixing study corrects with normal plasma and
Factor XI activity (%) 89 60- 140
who has normal factor IX levels is von Willebrand disease.
Ristocet in cofactor (IU/m l ) 041 0.50-1.5
CLOSE
Copynght © 2022 Massachusetts Medical Society. All rights reseived
(D You are in Demo mode. Results are not saved.
A 30-year-old woman who has been taking an oral contraceptive presents with In addition to stopping the oral contraceptive and recommending
swelling and tenderness in her left calf. An ultrasound reveals an acute alternative forms of contraception, which one of the following treatments is
t hrombus involving the left popliteal vein. She has normal renal function, is most appropriate for this patient?
taking no other medicat ions, and weighs 67 kg.
Not there yet...
75% Correct
I Start rivaroxaban 7 5 mg twice daily for 3 weeks and
then decrease the dose to 20 mg once daily
DETAILED FEEDBACK:
CLOSE
Copynght © 2022 Massachusetts Medical Society. All rights reserved
(D You are in Demo mode. Results are not saved.
A 45-year-old man who developed a left lower-extremity deep-vein thrombosis Which one of the following management approaches is most appropriate
after hi p replacement surgery 6 months ago presents for follow-up. He has for this patient?
done well on apixaban, with no bleeding complications, and has resumed
normal activities with no residual leg swelling or pain. :You got 1t1
Physical examination is entirely normal. One week ago, the patient had a
repeat ultrasound of t he left leg veins as part of a research study. The 52% Right Discontinue apixaban
ultrasound revealed a residual nonocclusive thrombus in his left common
femoral vein. 2% Other Discontinue apixaban and switch to dabigatran
6% Other
I Discontinue apixaban and check the D-dimer level; if
the D-dimer level is elevated, resume apixaban
DETAILED FEEDBACK:
CLOSE
Copynght © 2022 Massachusetts Medical Society. All rights reserved
(D You are in Demo mode. Results are not saved.
CLOSE
Copyright© 2022 Massachusetts Medical Society. All rights reserved.
(D You are in Demo mode. Results are not saved.
A 21-year-old woman with a one-year history of kidney disease caused by Which one of the following next steps is most appropriate in this patient's
postpartum hemolytic-uremic syndrome presents one day after revision of care?
her dialysis catheter. She reports continued oozing blood at the surgical site,
dizziness, and a "foggy head." She has been treated with erythropoietin for 6 Not there yet...
months.
Her temperature is 37.2°C, her heart rate is 68 beats per minute, and her blood 79% Correct Administer desmopressi n
pressure is 135/74 mm Hg. She is conversant. A few beats of asterixis are
observed. Her conjunctivae are clear, and there are no oral petechiae. Her Transfuse packed red cells to a target hematocrit of
4% Wrong
lungs are clear to auscultation, and the cardiac examination is normal. Her 35%
abdomen is soft without organomegaly. She has trace peripheral edema. Her
skin is clear, dry, and intact. Her dialysis catheter is intact, but t he insertion site Adm inister higher doses of erythropoietin to a target
8% Other
is encircled with oozing blood. hematocrit of 35%
Her laboratory results are as follows:
4% Other Administer conjugated est rogens
Patient value Reference range
Platelet count (per mm 3) 158,000 150,000-3 50,000 The most appropriate treatment for a patient with kidney fai lure and
Sodium (mEq/liter) 144 136-145 bleeding is to adm inister desmopressin.
CLOSE
Copynght © 2022 Massachusetts Medical Society. All nghts reserved
(D You are in Demo mode. Results are not saved.
While awaiting upper endoscopy, a 31-year-old woman with hematemesis and What error-prevention strategy should be emphasized in debriefing the
acute upper gastrointestinal bleeding receives a transfusion containing one medical team after this critical incident?
unit of packed red cells. Approximately 15 minutes into the transfusion, the
patient develops chills, fever, chest pain, and hypotension. You got itl
One of the nurses realizes that the administered blood product had been
intended for a t rauma patient with a similar last name. Check at least two patient identifiers before
89% Right
The transfusion is stopped, urgent therapy is given, and the blood bank is adm inistering a transfusion
notified.
Designate one health care provider to oversee the
4% Other
entire transfusion process from order to administration
Other
Ask the patient to identify his or her blood group before
3%
administering a t ransfusion
DETAILED FEEDBACK:
This patient's symptoms of fever, chest pain, and hypotension support the
diagnosis of an acute hemolytic reaction caused by the transfusion of ABO-
incompat ible blood. Most hemolytic reactions are caused by human error.
Therefore, in high-stakes sit uations, such as transfusion of blood products,
the standard of care is to verify the patient's identity using at least t wo
identifiers (such as name, medical record number, and date of birth listed
on wristband) before transfusing blood.
Limiting the blood-transfusion process to a single provider, from the initial
CLOSE
Copyright © 2022 Massachusetts Medical Society. All rights reserved
(D You are in Demo mode. Results are not saved.
A 34-year-old woman presents to the emergency department reporting 3 days What Is the most appropriate 1n1t1al therapy for this pat ient?
of progressive fatigue. She has a history of adenomyosis-related heavy
menstrual bleeding that has been controlled with continuous estrogen- Not there yet 11
Platelet count (per mm 3) 255,000 150,000-350,000 The most appropriate initial therapy for a patient who has heavy menstrual
bleeding with hemodynamically significant hemorrhage is red-blood-cell
The international normalized ratio is 1.1, and the partial thromboplastin t ime is transfusion.
28 seconds (reference range, 22.1-35. 1).
A complete blood count one year ago documented a hemoglobin DETAILED FEEDBACK:
concentration of 13 g/dL and a mean corpuscular volume of 86 µm 3
Heavy menstrual bleeding (HMB) affects 5% to 15% of reproductive-age
A urine pregnancy test is negative. females and can result from structural abnormalities such as adenomyosis,
endometrial polyps, and neoplasia, as w ell as nonstructural disorders such
as pelvic inflammatory disease and bleeding diatheses (including von
Willebrand disease).
Recurrent uncontrolled HMB uniformly results in iron-deficiency anemia,
marked by a hypoproliferative, hypochromic, microcytic anemia. Because
this is a chronic process, most affected individuals adapt to the anemia
physiologically over time and remain free of symptoms. Management for
these patients hinges on control of the menstrual bleeding and on iron
supplementation, which obviates the need for red-blood-cell (RBC)
transfusions.
Occasionally, patients present with more-brisk hemorrhage, resulting in
sub;-in tP htiquP ,md hPmodvmmir 1nst1bility ThPsP p;:i'IPnts n•nuIrP
CLOSE
Copynght © 2022 Massachusetts Medical Society. All nghts reserved
(D You are in Demo mode. Results are not saved.
A 45-year-old man presents with 2 days of dental pain from a tooth infection Which one of the following ant icoagulant management approaches is most
for which his dentist has recommended a dental extraction. He had a appropriate for t his patient?
spontaneous deep-vein thrombosis diagnosed in his left common femoral
vein 2 months ago and has been taking warfarin since then. (He has been Not there yet...
unable to afford a direct oral anticoagulant because he does not have health
insurance.) His international normalized ratio has been at target for the past 6
38% Correct Continue warfarin and proceed with the extraction
weeks, and the most recent value was 2.4.
DETAILED FEEDBACK:
CLOSE
Copyright © 2022 Massachusetts Medical Society. All rights reserved
(D You are in Demo mode. Results are not saved.
A 50-year-old woman with a history of myelodysplastic syndrome has What is the most appropriate next step in managing this patient's
received a bone-marrow transplant from an unrelated donor. Despite repeated thrombocytopenia?
transfusions of apheresis (single-donor) platelets, her platelet count has not
risen above 6000 per mm 3 (reference range, 150,000-350,000). Not t here yet...
On presentation, she is afebrile. Laboratory testing shows no evidence of
disseminated intravascular coagulation or thrombotic thrombocytopenic 26% Correct Perform human leukocyte antigen antibody testing
purpura.
29% Wrong Perform platelet-specific antibody testing
17% Other
I Observe the patient without initiating further platelet
transfusions
DETAILED FEEDBACK:
CLOSE
Copyright © 2022 Massachusetts Medical Society_ All rights reserved
(D You are in Demo mode. Results are not saved.
A 66-year-old woman presents to t he emergency department with a one-day Which one of the following tests is most likely to reveal the cause of this
history of bright-red blood in her stool. pat ient's bleeding diathesis?
During the past 3 months, she has noticed easy and excessive bruising on her
arm s and legs. She also reports three spontaneous nosebleeds in the past 2 Not there yet...
weeks. She reports progressive dyspnea for t he past year, as well as chest
pain and pressure on exertion in the past month. She denies excessive 64% Correct Von Willebrand multimer analysis
bleeding, nosebleeds, or easy bruising before the current episodes; is not
aware of a family history of bleeding disorders; has not sought medical care in 5% Wrong Factor VIII testing
more than 10 years; and takes no medications.
Her heart rate is 110 beats per minute, and her blood pressure is 90/75 mm 8% Other Mixing studies
Hg. Physical examination reveals a slow upst roke of the carotid pulse, a loud
systolic crescendo-decrescendo murmur, and multiple scattered 17% Other Von Willebrand factor antigen level
ecchymoses (of various ages) over her extremities.
6% Other Measurement of fibrinogen levels
Laboratory testing yields the following results:
Hemoglobin (g/dL) 9.8 12.0-76.0 Patients wit h aort ic stenosis may develop a bleeding diathesis t hat is
related to acquired von Willebrand syndrome.
Mean corpuscular volume (µm 3) 80 80-700
CLOSE
Copyright © 2022 Massachusetts Medical Society_ All nghts reserved
(D You are in Demo mode. Results are not saved.
A 45-year-old woman presents with new bruising and hematemesis. She has In addition to preparing the patient for an upper endoscopy, which one of
a history of cryptogenic cirrhosis and end-stage liver disease and was the following treatments is most appropriate for this patient?
discharged from the hospital one week ago after recovering from
spontaneous bacterial peritonit is. Her current medications include lactulose, Not there yet...
furosemide, spironolactone, and propranolol.
Her vital signs are normal. Her examination is notable for new large 43% Correct Cryoprecipitate
ecchymoses over her bilateral forearms and shins, abdominal ascites, and 2+
pitting lower-extremity edema. Her stool is black and tests positive on a feca l 37% Wrong Fresh frozen plasma
blood test.
Laboratory results are as follows 2% Other Platelet transfusion
Creatinine (mg/ dL) 2.0 0.6- 1.1 The most appropriate treatment for a patient with end-stage liver disease
and a low fibrinogen level who is bleeding is cryoprecipitate.
Fibrinogen (mg/dL) 65 150-400
CLOSE
Copyright © 2022 Massachusetts Medical Society. All rights reserved
(D You are in Demo mode. Results are not saved.
A 31-year-old man presents for follow-up one week after being diagnosed with If a decision is made to init iate testing at this visit, which of the following
deep-vein thrombosis (DVT) at an urgent-care clinic, where he presented with tests, if any, could provide accurate information?
pain and swelling in his right leg. He has been treated with apixaban since the
diagnosis. You got itl
The patient reports no recent travel or surgery Two years ago, he had a
similar episode involving the left leg and received anticoagulation for 6
35% Right
I Factor V Leiden and prothrombin gene-mutation
months. He notes that his mother and sister have also had episodes of DVT. testing
He is concerned about a genetic predisposition to venous thrombosis and
would like to be tested at this visit. 4% Other Prothrombin gene mutation only
The main tests that can detect a predisposit ion for venous
thromboembolism in a patient with a recent deep-vein thrombosis who is
taking anticoagulation are those for factor V Leiden and prothrombin gene
mutations.
DETAILED FEEDBACK:
CLOSE
Copyright © 2022 Massachusetts Medical Society_ All rights reserved
(D You are in Demo mode. Results are not saved.
A 65-year-old woman who is group B, Rh-positive requires urgent blood Which one of the following blood types can this patient safely receive?
transfusion. However, the blood bank has very low inventory for group B, Rh-
positive units. Not there yet. . 11
A person with type B, Rh-posit ive blood can receive compatible blood from
a donor with either B or O (Rh-positive or Rh-negative) blood.
DETAILED FEEDBACK:
The ABO blood group, the most immunogenic blood group, plays a role in
blood transfusion and solid-organ transplantation. The four common blood
groups in the ABO/ ABH system are 0, A, B, and AB.
The ABO antigens may present with four possible phenotypes - A, B, 0, or
AB - determined by t he presence or absence of two carbohydrate-based
ant igens, A or B, on the red cells' membranes. The ABO system is also
characterized by the presence or absence of naturally occurring ant ibodies
in the plasma against the missing antigens A and/or B.
A pat ient should receive a blood product compatible with his or her blood
group, as well as the preformed antibodies, as these antibodies can bind-
complement and cause an immediate hemolytic transfusion reaction.
Therefore, a patient with a B blood type can receive red cells from donors
who have either group B or group O blood, containing minimum
incompatible plasma. The patient cannot receive blood group A or AB, given
CLOSE
Copynght © 2022 Massachusetts Medical Society_ All nghts reserved
(D You are in Demo mode. Results are not saved.
A 34-year-old woman with sickle cell disease who receives monthly red-cell Which one of the following treatments is most appropriate for managing
transfusions to prevent sickling crises presents with fatigue. this patient's condition?
Her blood pressure is 700/50 mm Hg, her heart rate is regular and 60 beats
per minute, her temperature is 37°C, and her oxygen saturation is 98% while Not there yet...
she is breathing ambient air.
Laboratory results are as follows: 80% Correct Deferasirox
DETAILED FEEDBACK:
CLOSE
Copyright © 2022 Massachusetts Medical Society. All rights reserved
(D You are in Demo mode. Results are not saved.
A 78-year-old man with amyloidosis receives t wo units of packed red cells as Which one of the following approaches could have prevented this
treatment for severe symptomat ic anemia. After 350 ml are infused, he transfusion reaction?
develops shortness of breath and cough. His blood pressure increases from
120/65 mm Hg to 165/90 mm Hg, his temperature remains at 37.0°C, his Not there yet...
heart rate increases from 65 to 104 beats per minute, and his oxygen
saturation drops to 85% while he is breathing ambient air. Supplemental
56% Correct Administering blood slowly
oxygen is provided, but t he patient has an inadequate response and
endotracheal intubation is required.
10% Wrong Irradiating the blood units
Physical examination reveals bulging neck veins.
A chest radiograph is obtained (figure). 6% Other Adm inistering blood from an Rh-negative donor
The correct diagnosis for a patient who develops acute respiratory dist ress
and evidence of heart failure after receiving a red-cell transfusion is
transfusion-associated circulatory overload.
DETAILED FEEDBACK:
This pat ient had known amyloidosis and despite the absence of symptoms
of heart failu re, the heart stiffness from amyloid infiltration dramatically
reduced his physiologic reserve. Small increases in intravascular volume
can precipitate acute decompensation and heart fai lure. In most settings of
transfusion, more units are transfused before one precipitates heart failure.
Older patients, children, and patients with renal or cardiac fai lure are at
increased risk for transfusion-associated circulatory overload (TACO).
TACO is defined as new onset or exacerbation of at least three of the
following condit ions within 6 hours after completion of the transfusion
• Acute respiratory distress (dyspnea, orthopnea, cough)
• Evidence of positive fluid balance
• Elevated brain natriuretic peptide level
CLOSE
Copynght © 2022 Massachusetts Medical Society. All rights reserved
(D You are in Demo mode. Results are not saved.
A 65-year-old woman being treated with induction chemotherapy for acute Which one of the following management approaches is most appropriate
myeloid leukemia develops chills and a 1.2°C increase in body temperature, for th is patient?
without any shortness of breath or change in blood pressure, 2 hours after
receiving one unit of group B, Rh-negative packed red cells. Her blood group is Not there yet...
B, Rh-positive; she did not receive any premedication.
After she receives 650 mg of acetaminophen, her fever subsides. A c lerical 36% Correct Perform a direct antiglobulin test
check confirms she received the correct unit of blood; retesting of a sample
from the transfused blood showed that it had been typed correctly. 2% Wrong Obtain a chest radiograph
The most appropriate test for a patient who becomes febrile after receiving
a correctly matched unit of packed red cel ls is a postreaction direct
antiglobulin test to rule out a hemolytic transfusion reaction.
DETAILED FEEDBACK:
CLOSE
Copyright © 2022 Massachusetts Medical Society. All nghts reserved
(D You are in Demo mode. Results are not saved.
DETAILED FEEDBACK:
CLOSE
Copynght © 2022 Massachusetts Medical Society. All rights reserved
G) You are in Demo mode. Results are not saved.
A 33-year-old woman who is at 30 weeks' gestation w ith her first pregnancy Which one of the following management approaches is most appropriate
presents w ith new-onset dyspnea. for this patient?
Her physical examination is notable for a slight fever, tachycardia, and a
moderately swollen left leg Laboratory testing reveals a leukocyte count of Not there yet...
6500 per mm 3 (reference range, 4500-11,000)
A lower-extremity compression ultrasound is positive for a deep-vein Start subcutaneous low-molecular-weight heparin and
65% Correct
thrombosis in the left femoral vein. discontinue it 24 hours before delivery
DETAILED FEEDBACK:
CLOSE
Copyright © 2022 Massachusetts Medical Society_All nghts reserved
(D You are in Demo mode. Results are not saved.
A 50-year-old woman has an externalized central venous cat heter (Hickman What, if any, anticoagulation therapy is appropriate to prevent catheter-
catheter) placed on her right chest wall so that she can receive chemotherapy associated thrombosis in this patient?
for recently diagnosed breast cancer.
Not there yet...
5% Other
I Unfractionated heparin 5000 units subcutaneously
twice daily
DETAILED FEEDBACK:
CLOSE
Copyright © 2022 Massachusetts Medical Society_All rights reserved
(D You are in Demo mode. Results are not saved.
A 67-year-old man presents with palpitations and is diagnosed with Which one of the following medications is most appropriate for reducing
nonvalvular atrial fibrillation. He has a history of well-controlled type 2 this patient's risk for thromboembolism?
diabetes, hypertension, and stage 5 chronic kidney disease but is not on
dialysis. Not there yet...
His serum creatinine level is 4.3 mg/dl (reference range, 0.8-1.3), with an
estimated glomerular filtrat ion rate of 13 ml/min/1 .73 m 2 (2:60). The results 80% Correct Warfarin
of his liver function tests are within normal limits. He has no history of
bleeding. 6% Wrong Dabigatran
3% Other Aspirin
9% Other Rivaroxaban
DETAILED FEEDBACK:
CLOSE
Copynght © 2022 Massachusetts Medical Society. All nghts reserved
(D You are in Demo mode. Results are not saved.
A 70-year-old woman presents to the emergency department with back and In addition to discontinuing the apixaban, establishing peripheral vascular
flank pain and a recent syncopal episode. She reports that she developed access, and implementing initial resuscitative measures, which one of the
subacute onset of back pain radiating around her left flank this morning. She fol lowing management approaches is most appropriate in this patient's
also developed worsening fatigue until she eventually had a syncopal episode care?
after standing up from a seated position. Her medical history includes
nonvalvular atrial fibril lation, which has been treated with apixaban for the Not there yet...
past 9 months. She took her last dose of this medication 5 hours ago.
On examination, her blood pressure is 80/60 mm Hg (standing) with a heart 60% Correct Administer andexanet alfa
rate of 112 beats per minute. There is vague tenderness over her flank and
back. 17% Wrong Administer fresh frozen plasma
Laboratory evaluation shows that her hemoglobin level has declined from a
baseline of 13.0 g/dl to 9.7 g/dl (reference range, 12.0- 16 0), and her 5% Other Administer recombinant factor VIia
hematocrit has decl ined from 40% to 29% {36-46) during the past 4 weeks.
14% Other Administer idarucizumab
A CT scan of the abdomen shows that she has a retroperitoneal hematoma.
There is no abdominal aortic aneurysm or other findings.
4% Other Administer desmopressin
DETAILED FEEDBACK:
CLOSE
Copyright © 2022 Massachusett s Medical Society. All nghts reserved
(D You are in Demo mode. Results are not saved.
lmmunoglobulin M (mg/dL) 5000 53-334 The most likely diagnosis for a patient w ith features of hyperviscosity who
is found to have plasmacytoid lymphocytes in the bone marrow is
Serum viscosity (centipoise) 7.0 14-1.8
Waldenstrom macroglobulinemia.
A bone-marrow biopsy demonstrates 30% infiltration by small plasmacytoid
lymphocytes. DETAILED FEEDBACK:
CLOSE
Copyright © 2022 Massachusetts Medical Society_ All rrghts reserved
(D You are in Demo mode. Results are not saved.
A 67-year-old man is concerned about his family history of leukemia. His Which one of the following screening programs is most appropriate, given
younger brother was diagnosed with acute myeloid leukemia at age 52. After this patient's family history?
induction chemotherapy, the brother entered remission but relapsed at age 53;
despite salvage chemotherapy, he died of infectious complications at age 54. Not there yet...
The patient also has a second cousin who may have a history of leukemia.
The patient's previous provider recommended t hese acute myeloid leukemia Routine medical care without additional specific
81% Correct
screening tests: a complete blood count with differential every 3 months and screening
flow cytometry of the peripheral blood annually. The results have been normal.
Annual molecular testing of peripheral blood for FMS-
5% Wrong
like tyrosine kinase 3 mutation
The best screening for a patient who has a sibling with a history of acute
myeloid leukemia is routine medical care.
DETAILED FEEDBACK:
Patients who have siblings with a history of acute myeloid leukemia (AML)
are often anxious about t heir own risk after witnessing the precipitous
course of this illness. Families with clusters of AML are rare, and the
preponderance of data does not suggest a highly elevated risk for siblings
of affected patients.
Situations that may suggest an increased risk include family pedigrees with
multiple affected individuals and siblings of pat ients diagnosed before age
21 . Even in such patients, however, routine medical care is the only
accepted screening program for AML. Patients whose family members with
AML have a known germ li ne mutation are best evaluated within a specialty
center with appropriate genet ic counseling.
Costly AML screening oroqrams without known benefit - including
CLOSE
Copynght © 2022 Massachusetts Medical Society. All rights reserved
(D You are in Demo mode. Results are not saved.
A 52-year-old man presents with new-onset fatigue and abdominal fu llness. What Is the most likely diagnosis in this case?
He reports intermittent fevers and a 4.5-kg weight loss in the past 2 months.
He works as a lawyer and leads an active lifestyle, but for the past several Not there yet .. 11
Lymphocytes 25 22-44
DETAILED FEEDBACK:
Monocytes 2 4-11
Hairy-cell leukemia is an uncommon hematologic cancer characterized by
Atypical lymphoid cells 38 -
an accumulation of abnormal B lymphocytes. It is usually classified as a
The atypical lymphoid cells are described as having radial projections. subtype of chronic lymphoid leukemia. Hairy-cell leukemia makes up
approximately 2% of al l leukemias.
Hairy-cell leukemia can manifest, in advanced stages, with B symptoms,
pancytopenia, and marked splenomegaly. Monocytopenia is also a
common finding. Pancytopenia occurs in relation to several
clinicopathological findings, including infiltration of the bone marrow by
hairy-cell leukemia cells, splenic sequestration of blood cells because of
massive splenomegaly, and bone-marrow fibrosis. In most cases of hairy-
cel l leukemia, periphera l-blood specimens reveal atypical B lymphocytes
with radial hairy projections. In patients with hairy-cell leukemia who do not
have lymphocytes with the typical hairy project ions, the diagnosis may rest
on documenting high levels of soluble CD25 in the serum and detecting cel l
CLOSE
Copyright © 2022 Massachusetts Medical Society. All nghts reserved
(D You are in Demo mode. Results are not saved.
CLOSE
Copynght © 2022 Massachusetts Medical Society_ All nghts rese1Ved
(D You are in Demo mode. Results are not saved.
The most likely diagnosis in a young man with severe anemia, a low
reticulocyte count, and a lack of erythroid precursors on bone-marrow
examination is acquired pure red-cell aplasia.
DETAILED FEEDBACK:
CLOSE
Copyright© 2022 Massachusetts Medical Society. All rights reserved.
(D You are in Demo mode. Results are not saved.
A 55-year-old woman presents for her annual physical examination. She feels Which one of the following management strategies is most appropriate for
well and is physically active. this patient?
Laboratory testing yields the following results:
Not there yeL
Patient value Reference range
Leukocyte count (per mm3) 4600 4500-11 ,000 78% Correct Surveillance
Hemoglobin (g/dL) 14.2 12.0-16.0
4% Wrong Autologous bone-marrow transplant
Platelet count (per mm 3) 152,000 150,000-350,000
CLOSE
Copynght © 2022 Massachusetts Medical Society_All nghts rese1Ved
(D You are in Demo mode. Results are not saved.
A 34-year-old woman presents for follow-up of an episode in which she lost Which one of the following laboratory tests is most likely to be elevated in
consciousness in a dentist's office after taking penicillin prophylactically in this patient?
anticipation of a tooth abscess extraction. She was found to be hypotensive
and was resuscitated by paramedics using epinephrine, intravenous fluids, Not there yet...
and oxygen . The episode was preceded by a 3-month history of recurrent
flushing.
55% Correct Serum tryptase level
Her medical history is otherwise unremarkable except for a mildly pruritic skin
rash that she has noticed for the past few years; she has not sought medical 4% Wrong Plasma and urine metanephrine levels
attention for t his. She has no major allergies or any recent spider bites, bee
stings, or other exposures to venom. 17% Other Cl esterase inhibitor level
On examinat ion, she is afebrile. Her heart rate is 84 beats per minute, her
blood pressure is 134/82 mm Hg, and her oxygen saturation is 97% while she 11% Other 24-hour urine sample for 5-hydroxyindoleacetic acid
is breathing ambient air. Her lungs are clear to auscultation bilaterally, and her
heart is regular. Abdominal examination reveals no hepatosplenomegaly. 13% Other Serum and 24-hour urine testing for porphyrins
Red-brown maculopapular skin lesions, some of which darken and swell when
rubbed, are present on the t runk and bilateral inguinal reg ions (fi gure).
KEY LEARNING POINT:
Laboratory results are as follows:
The presence of red-brown maculopapular skin lesions, some of which
Patient value Reference range
darken and swell when rubbed, in an individual with a history of flush ing
Leukocyte count (per mm3) 6300 4500-11,000 and penicillin-induced anaphylaxis suggests a diagnosis of mastocytosis.
CLOSE
Copynght © 2022 Massachusetts Medical Society. All nghts reserved
(D You are in Demo mode. Results are not saved.
A 30-year-old woman is hospitalized with a 3-week history of progressive Which one of the following next steps Is most appropriate for this patient?
dyspnea on exertion, intermittent fevers, night sweats, and cough. She says
she has no headaches or vision changes. Not there yet . 1·
Total bilirubin (mg/dL) 04 0.3- 1.0 The first step in the management of a pat ient who is hemodynamically
Creatinine (mg/dL) 1.1 0.6-1.1 stable and has symptoms of superior vena cava syndrome and a
mediastinal mass is to obtain tissue diagnosis via biopsy.
Lactate dehydrogenase (U/ liter) 634 100-190
CT of t he chest, abdomen, and pelvis reveals a 15-cm by 10-cm mediastinal DETAILED FEEDBACK:
mass, w it h moderate compression of the superior vena cava and no evidence
This patient has an isolated bulky mediastinal mass, an elevated lactate
of tracheal compression or venous t hrom bosis. There are no pleural or
dehydrogenase level, and B sym ptom s (fever, unintentional weight loss, and
pericardia! effusions. There are no other sites of lymphadenopathy.
night sweats). This presentation is typical of primary m ediastinal large B-
cell lym phoma. An accurate histologic diagnosis is essential prior to
starting therapy, even before starting glucocorticoids, given that
glucocorticoids can adversely affect an accurate pathologic diagnosis or
cause a delay in a defi nitive diagnosis.
Al though this patient has compression of the superior vena cava, there is
no evidence of thrombosis, so thrombolytic t herapy or heparin are not
indicated.
There is a limited role for placing an endovascular stent in the superior vena
cava or irradiat ing the mediast inal mass emergently if the patient does not
have severe respiratory comprom ise or neurologic changes.
CLOSE
Copyright © 2022 Massachusetts Medical Society. All rights reserved
(D You are in Demo mode. Results are not saved.
A 75-year-old previously healthy man complains of slowly progressive fatigue In addition to transfusion support, which one of the following treatments is
and exercise intolerance during the past 6 months. He has not noticed any the most appropriate initial treatment for this patient?
bleeding.
On examination, his vital signs are normal apart from a regular heart rate of Not there yet...
115 beats per minute. He appears fat igued and pale. His lung and abdominal
exams are unremarkable. He has trace ankle edema. There are no petechiae 54% Correct Lenalidomide
or ecchymoses.
Laboratory testing reveals the following results: 16% Wrong Allogeneic hematopoietic stem-cell transplant
- -
Patient value Reference range Other
7% Decitabine
Leukocyte count (per mm3) 4000 4500-11,000
12% Other Induction chemotherapy
Neutrophils (per mm 3) 1500 1900-7600
Furt her laboratory workup for blood loss, hemolysis, and nutrient deficiency is
DETAILED FEEDBACK:
unremarkable. A bone-marrow aspirate and biopsy are consistent with a
myelodysplastic syndrome. The Sq minus subtype of myelodysplastic syndrome (MOS) is
Cytogenetics shows a deletion of chromosome 5q, consistent wit h a characterized by chronic transfusion-dependent anemia, wh ich can lead to
diagnosis of t he Sq minus syndrome subtype of myelodysplast ic syndrome. iron overload. The characteristic chromosome abnormality is an interstitial
deletion involving the long arm of chromosome 5 (usually 5q31q33). An
elevated platelet count can be a clue to this subtype of MOS in some
pat ients. Neutropenia or t hrombocytopenia is rare, and the risk for
progression to acute myeloid leukemia is low.
Lenalidomide is the most appropriate first-line treatment for the Sq minus
syndrome. About two-thirds of affected patients achieve transfusion
independence, and about half have a complete cytogenetic response.
Azacitidine and decitabine are hypomethylating agents that could also be
considered. However, these agents have a higher risk-benefit ratio in
pat ients with low-risk disease. They are also generally reserved for patients
l."'ith hiaher rislt MOS or lower risf ri;itIents v, ho r::innnt toler-:ite resonnd
CLOSE
Copyright © 2022 Massachusetts Medical Society. All nghts reserved
(D You are in Demo mode. Results are not saved.
A 61-year-old man reports episodic burning and erythema in his hands for the Which one of the following diagnoses Is most likely In this case?
past several weeks. The episodes, which last from minutes to several hours
and are not prompted by cold temperature, have been increasing in frequency Not there yet .. I
for 2 weeks. The patient also reports epigastric pain, headache, fatigue, and
diffuse pruritus. He used to drink one or two beers per day, but he stopped
84% Correct Polycyt hemia vera
because alcohol consumption sometimes triggered an episode. He quit
smoking 8 years ago.
2% Wrong Obstructive sleep apnea
On examination, the patient has a heart rate of 72 beats per minute, a blood
pressure of 127/78 mm Hg, and a BMI of 22. He has notable facial plet hora. 9% Other Essential thrombocythemia
His spleen is palpated 4 cm below the left costal margin. He has no
hepatomegaly or palpable lymphadenopathy. 3% Other Renal-cell carcinoma
Laboratory testing yields the fol lowing results:
2% Other Carbon monoxide exposure
Patient value Reference range
Leukocytes (per mm3) 14,300 4500-11,000 A patient with erythromelalgia, an elevated hemoglobin level, and a low
serum erythropoietin level is likely to have polycythemia vera.
Platelets (per mm 3) 787,000 150,000-350,000
CLOSE
Copyright © 2022 Massachusetts Medical Society. All nghts reserved
(D You are in Demo mode. Results are not saved.
A 70-year-old woman presents with slowly progressive fatigue and dyspnea. Which one of the following diagnoses Is most likely In this case?
Her medical history is significant for breast cancer that was diagnosed 5
years ago and t reated with lumpectomy followed by adJuvant radiotherapy Not there yet... 11
CLOSE
Copynght © 2022 Massachusetts Medical Society. All nghts reserved
(D You are in Demo mode. Results are not saved.
Leukocyte count (per m m 3) 4000 4500- 11,000 The most appropriate first-line treatments for symptomat ic anemia in
Hemoglobin (g/dL) 8.8 12.0-16.0 patients with low-risk myelodysplastic syndrome are erythropoietin and
transfusion support.
Mean corpuscu lar volume (µm 3) 100 80-700
CLOSE
Copyright © 2022 Massachusetts Medical Soc1ety. All nghts reserved.
(D You are in Demo mode. Results are not saved.
A 75-year-old man reports a 4-month history of progressively worsening Which one of the following diagnoses Is most likely In this case?
dyspnea, fatigue, and exercise intolerance. One week ago, he noticed
unexplained bruising and had two episodes of epistaxis. He has smoked one Not there yet .. II
pack of cigarettes daily for 55 years.
On examination, the patient is afebrile, in no distress, and appears pale and 72% Correct I Myelodysplastic syndrome
tired. His heart rate is 105 beats per minute, his blood pressure is 110/70 mm
Hg, and his respiratory rate is 18 breaths per minute. He has normal oxygen 7% Wrong Chronic myelomonocytic leukemia
saturation, his heartbeat is regular and tachycardic, and his lungs are clear.
His abdomen is soft without hepatosplenomegaly. He has several 12% Other Aplastic anemia
ecchymoses on his extremities and a petechial rash on his lower extremities,
but no edema. 5% Other Clonal hematopoiesis of indeterminant potential
Laboratory testing reveals these results:
5% Other Acute myeloid leukemia
Patient value Reference range
Monocytes (per mm 3) 500 100-800 The findings of pancytopenia, bone-marrow dysplasia, and a monosomy 7
clone on cytogenetic analysis establish a diagnosis of myelodysplast ic
Hemoglobin (g/dL) 7.2 13.5-17.5 syndrome.
Mean corpuscular volume (µm 3) 104 80-100
Total bilirubin (mg/dL) 0.9 0.3-1.0 The myelodysplastic syndromes (MOS) are a heterogeneous group of
clonal bone-marrow neoplasms characterized by ineffective hematopoiesis
Creatinine (mg/dL) 0.5 0.8-1.3
and risk for progression to acute myeloid leukemia. The diagnosis should
A bone-marrow aspirate and biopsy shows 90% cellularity with trilineage be suspected in an older patient with cytopenias and can be confirmed by
dysplasia. Myeloid blasts are not increased. Cytogenetic analysis reveals a bone-marrow biopsy. In patients with pancytopenia, trilineage dysplasia,
monosomy 7 clone. Testing for pathogenic variants in the BCR-ABL, JAK2, monosomy 7, and a myeloblast count <20%, the correct diagnosis is MOS.
CALR, and MPL genes is negative. In acute myeloid leukemia, the marrow myeloblast count is 2:20%.
Aplastic anemia is not typically associated with any chromosome
abnormalities or with bone-marrow hypercellularity.
Chronic myelomonocytic leukemia is a neoplasm with both
myelodysplastic and myeloproliferative featu res that presents with
dysplasia and a peripheral blood monocytosis of at least 1000 per mm 3.
However, monocytosis can also be seen in MOS without a diagnosis of
rhrnni, mv, lorn r 'VtI, leul<Pmi
CLOSE
Copyright© 2022 Massachusetts Medical Society_ All nghts rese1Ved
(D You are in Demo mode. Results are not saved.
A 76-year-old woman reports a 2-month history of worsening fatigue and Which one of the following diagnoses Is most likely In this case?
abdominal pain. She states that she has not had any fevers, chills, or recent
infections. Not there yet. II
On examination, she has splenomegaly and scattered ecchymoses on her
lower extremities. 58% Correct Chronic myeloid leukemia
Laboratory results from both today and 6 months ago are as follows:
4% Wrong Leukemoid reaction
Patient va lue 6 Current patient Reference range
months ago value
7% Other I Acute promyelocytic leukemia
Leukocyte count 15,300 135,500 4500-1 1,000
(per mm 3) 21% Other I Acute myeloid leukemia
Neutrophi ls (%) 60 69 40-70
10% Other I Chronic lymphocytic leukemia
Lymphocytes (%) 75 8 22-44
Platelet count (per 468,000 549,000 1 50,000-350,000 Chronic myeloid leukemia (CML) manifests with leukocytosis and a
mm 3) leukocyte differential that reflects the entire spectrum of the myeloid series.
- -- Absolute basoph il ia, eosinophilia, or both are often present.
Thrombocytosis and splenomegaly are also often present. Although many
patients are asymptomatic at presentation, typical symptoms include
fatigue, malaise, weight loss, night sweats, and abdominal fullness or
discomfort. Patients with <10% myeloblasts in the blood and bone marrow
have chronic-phase CM L. In contrast, patients with accelerated-phase CML
have a blast count of 10% to 20%, and patients with CML in blast crisis (e.g.,
acute myeloid leukemia) have >20% blasts in the peripheral blood or bone
marrow.
A leukemoid reaction, which usually occurs in response to an infection, is
characterized by a leukocytosis with neutrophilia and a left shift. It is
unusual to see cells more immature than metamyelocytes in leukemoid
CLOSE
Copyright © 2022 Massachusetts Medical Society. All nghts reserved
(D You are in Demo mode. Results are not saved.
A 70-year-old woman reports a one-month history of dizziness, headaches, Which one of the following tests is most appropriate to establish the cause
and paresthesias in her hands. She has hypertension but no other of this patient's elevated hemoglobin level?
comorbidities and has never smoked.
On examination, her heart rate is 75 beats per m inute, her blood pressure is You got itl
162/85 mm Hg, her oxygen saturation is 98% while she is breathing ambient
air, and her BMI is 22. Her lungs are clear on auscultation. An abdominal 76% Right Peripheral blood screening for mutations in JAK2
examination reveals splenomegaly, and examination of the hands reveals
erythema. 3% Other Serum ferritin level
Laboratory testing yields the following results:
13% Other Bone-marrow biopsy
Patient value Reference range
Leukocyte count (per mm3) 14,500 4500-11,000 3% Other Serum iron level and total iron-binding capacity
DETAILED FEEDBACK:
This patient most likely has polycythemia vera (PV), given the very elevated
hemoglobin level in combination wit h the low erythropoietin level. PV is an
acquired myeloproliferative neoplasm characterized by an increased
erythrocyte mass. The World Health Organization criteria for diagnosis
(updated in 2016) are as follows:
Major Criteria:
Elevated hemoglobin level (>16.0 g/ dL in women; >1 6. 5 g/ dL in men),
elevated hematocrit (>48% in women; >49% in men), or elevated red-
cell mass
2 Bone-marrow biopsy consistent with PV
3 Presence of the JAK2 V617F or another similar JAK2 mutation
Minor Criterion
CLOSE
Copyright © 2022 Massachusetts Medical Society_All rights reserved
(D You are in Demo mode. Results are not saved.
An 81-year-old woman has a routine complete blood count taken, revealing a Which one of the following management approaches is most appropriate
platelet count of 1,200,000 per mm 3 (reference range, 150,000-350,000), a for this patient's essent ial t hrombocythemia?
normal leukocyte count. and a norm al hemoglobin concentration. She has no
notable symptoms and no history of bleeding, myocardial infarction, stroke, or Not there yet...
deep-vei n thrombosis. She has generally been in good healt h and has no
history of tobacco use.
Hydroxyurea and measurement of ristocetin cofactor
66% Correct
Genetic testing documents positivity for the JAK2 V617F m utation. Bone- activity
marrow biopsy reveals megakaryocyte hyperplasia without other
abnormalities. 11% Wrong Low-dose aspirin alone
7% Other Plateletpheresis
DETAILED FEEDBACK:
CLOSE
Copynght © 2022 Massachusetts Medical Society_ All nghts reserved
(D You are in Demo mode. Results are not saved.
A 74-year-old woman reports fatigue and abdominal discomfort for the past Which one of the fo llowing diagnostic tests should this patient undergo
several months, as well as early satiety and 4.5 kg of unintended weight loss next?
during the past 6 months. She states that she has had no fevers or night
sweats. She was previously in excellent health. Not there yet...
On examinat ion, the patient has a heart rate of 85 beats per minute and a
blood pressure of 117/64 mm Hg. She exhibits conjunctiva! pallor but no 70% Correct Bone-marrow biopsy
palpable lymphadenopathy. No jugular venous distention is evident Her lung
examination is normal. Her spleen is palpated 14 cm below the left costal 16% Wrong Peripheral-blood flow cytometry
margin, and her liver is palpated 8 cm below the right costal margin. She has
no pedal edema. Positron emission tomography/ CT of t he chest,
5% Other
Laboratory testing yields the fol lowing results: abdomen, and pelvis
Lactate dehydrogenase (U/liter) 547 100-190 The most likely diagnosis in a patient with splenomegaly, anem ia, teardrop-
shaped erythrocytes, nucleated erythrocytes, and immature granulocytes is
A peripheral-blood smear reveals anisocytosis with numerous teardrop- primary myelof1brosis.
shaped and nucleated erythrocytes. The leukocyte differential includes 20%
neutrophils, 14% bands, 21 % metamyelocytes, 17% myelocytes, 3%
DETAILED FEEDBACK:
promyelocytes, and 2% blasts.
Primary myelofibrosis is a myeloproliferative neoplasm that results in
marrow fibrosis and extramedullary hematopoiesis A leukoerythroblastic
peripheral smear with nucleated erythrocytes, dacryocytes (teardrop-
shaped cells), immature myeloid precursors, and occasional circulating
blasts is a common finding.
Confirming a diagnosis of primary myelofibrosis requires a bone-marrow
biopsy, which typically reveals fibrosis that is evident from deposits of
reticulin, collagen, or both. The JAK2 V617F mutation is identified in 50% to
60% of patients with primary myelofibrosis. Primary myelofibrosis is a
diagnosis of exclusion, so secondary causes of marrow fibros is or
leukoerythroblastosis (e.g., tumors metastatic to the bone marrow,
granulomas, and autoimmune conditions) must be ruled out. Chronic
CLOSE
Copyright © 2022 Massachusetts Medical Society. All nghts reserved
(D You are in Demo mode. Results are not saved.
Platelet count (per mm 3) 90,000 150,000-350,000 Hemolytic anemia, a history of portal-vein thrombosis, and pancytopenia
together suggest a diagnosis of paroxysmal nocturnal hemoglobinuria
Reticulocyte count(% red cells) 6.0 0.5-2.5
(PNH).
Haptoglobin (mg/dL) <8 16-199
PNH is a rare, acquired bone-marrow disorder caused by mutations in the
Creatinine (mg/dL) 0.8 0.8-1.3 phosphatidylinositol glycan anchor gene, leading to the partial absence or
complete absence of glycosylphosphatidylinositol-linked proteins (e g.,
Total bilirubin (mg/dL) 2.5 0.3-1 0
CD55 and CD59) on blood cells. PNH cells are susceptible to complement-
Direct bilirubin (mg/d L) 0.3 0 1-0.3 mediated lysis that occurs in the intravascular space and causes
Lactate dehydrogenase (U/liter) 472 100-190 hemoglobinuria/ hemosiderinuria and episodes of dark urine.
Thrombosis, the most common cause of death in patients with PNH, may
Prothrombin t ime, partial-thromboplastin t ime, and D-dimer are within normal occur in unusual sites. Some patients w ith PNH experience bone-marrow
limits. A peripheral-blood smear reveals normal platelets, some macrocytic fa ilure resulting in pancytopenia. PNH is associated with increased risk for
red cells, and polychromasia. Urinalysis shows 2 red cells per high-power field myelodysplastic syndrome, aplastic anemia, and leukemia.
(reference range, 0-2) and no leukocytes. A direct ant iglobulin (Coombs) test
First-line therapy for hemolytic PNH is eculizumab, a monoclonal antibody
is negative.
CLOSE
Copynght © 2022 Massachusetts Medical Society. All nghts reserved.
(D You are in Demo mode. Results are not saved.
A 19-year-old woman is admitted to the intensive care unit for treatment of Which one of the following InterventIons should be 1nit1ated at this time?
acute lymphoblastic leukemia. Laboratory findings on admission are as
follows: Not there yet . II
Uric acid (mg/dl) 7.6 1.5- 6.0 Change the normal saline to a sodium bicarbonate
5% Other
infusion
Lactate dehydrogenase (U/liter) 894 100-190
Leukocytes (per mm 3) 94,900 4500-11,000 Tumor lysis syndrome (TLS) is caused by release of the contents of
Potassium (mEq/liter) 54 3.5-5.0 malignant cells into the bloodstream, resulting in electrolyte abnormalities,
which may include hyperuricemia, hyperkalemia, hyperphosphatemia, and
Creatinine (mg/dL) 1.9 0.6-1.1
hypocalcemia.
Uric acid (mg/dL) 10.9 1.5-6.0
Strategies for preventing and monitoring TLS vary depending on individual
Calcium (mg/dl) 8.0 9.0-10.5 risk for TLS. Risk varies depending on the tumor being treated: smaller solid
cancers and indolent lymphomas are associated with the lowest risk, and
Phosphorus (mg/dl) 6.0 3.0- 4.5
aggressive leukemias and cancers with large tumor burdens are associated
Intravenous furosemide is added to the patient's regimen and titrated to a goal with the highest risk. Patients with acute lymphocytic leukemia and a
urine output of 200 ml per hour. leukocyte count >50,000 cells per mm 3 (as in this case) are in the highest
risk category, and aggressive efforts at prevention would be advised.
When the risk of TLS is low, volume expansion during chemotherapy and
close 19borqtorv monitorinq is •voirqlly qn 'ldequilte milmiqement str::iteov
CLOSE
Copynght © 2022 Massachusetts Medical Society. All rights reserved
(D You are in Demo mode. Results are not saved.
A 70-year-old woman presents with back pain. She was recently diagnosed Which one of the following additional medications is most appropriate for
with a TS vertebral fracture and immunoglobulin G kappa multiple myeloma this patient?
that is complicated by diffuse skeletal lesions.
She has a creatinine level of 1.9 mg/dl (reference range, 0.6-1. 1) and a Not there yet...
calculated glomerular filtration rate of 28 ml/min/1.73 m 2 (::!60). Her serum
calcium and vitamin D levels are within normal limits. 51 % Correct Denosumab
Treatment with dexamethasone and bortezomib is started.
12% Wrong Calcium supplement with vitamin D
7% Other Calcitonin
4% Other Cinacalcet
A patient with multiple myeloma and lytic bone lesions should be treated
with an osteoclast inhibitor, such as an intravenous bisphosphonate or
denosumab.
DETAILED FEEDBACK:
CLOSE
Copynght © 2022 Massachusetts Medical Society_ All rights reserved
(D You are in Demo mode. Results are not saved.
A 74-year-old man with chronic lymphocytic leukemia is hospitalized w ith In addition to giving appropriate antibiotic coverage for pneumonia, which
fever, shortness of breath, and a productive cough. It is his t hird one of the following management approaches is most appropriate for this
hospitalization for pneumonia in the past year. He recently received treatment patient?
for chronic lymphocytic leukemia with fludarabine and rituximab. He has
received both the 23-valent pneumococcal polysaccharide vaccine and the You got itl
13-valent pneumococcal conjugate vaccine within the past 3 years.
Physical examination reveals palpable splenomegaly. A chest radiograph 62% Right Intravenous immunoglobulin
reveals a right-lower-lobe infiltrate.
Laboratory findings are as follows: 9% Other Splenectomy
DETAILED FEEDBACK:
CLOSE
Copyright © 2022 Massachusetts Medical Society. All nghts reserved
(D You are in Demo mode. Results are not saved.
A 70-year-old man presents for follow-up after laboratory testing revealed an Which one of the following management approaches is most appropriate
elevated total protein level of 9.0 g/dl (reference range, 5.5-8.0) and an for th is patient?
albumin level of 3.4 g/dl (3.5-5.5). He states that he does not experience
bone pain, fevers, night sweats, weight loss, fatigue, numbness, or dizziness. :You got 1t'
Physical examination is unremarkable.
On further laboratory testing, the hemoglobin, leukocyte count, platelet count, 75% Right Schedule periodic laboratory monitoring
calcium level, and serum creatinine level are normal. Serum protein
electrophoresis shows a monoclonal protein measuring 1.4 g/dl, and 7% Other Perform fat-pad biopsy for amyloid
immunofixation shows this band to be immunoglobulin G kappa. The serum
free-light-chain-kappa-to-lambda ratio is 1.90 (reference range, 0.26-1.65). 7% Other Repeat bone-marrow biopsy in one year
Urine protein electrophoresis and immunofixation show no monoclonal
protein. A bone survey shows no lytic bone lesions or fractures. A bone- 6% Other Initiate treatment with dexamethasone and bortezomib
marrow biopsy and aspirate reveal 6% plasma cells.
5% Other Perform bone scan
DETAILED FEEDBACK:
CLOSE
Copynght © 2022 Massachusetts Medical Society. All rights reserved
(D You are in Demo mode. Results are not saved.
A 35-year-old woman with common variable immunodeficiency treated with What Is t he most likely d1agnos1s in this case?
intravenous im munoglobulin once monthly for 3 months presents with night
sweats and a 6.8-kg weight loss during the past few months. A system s Not there yet.. I
review is otherwise unremarkable.
The patient is afebri le but has a chain of enlarged cervical lymph nodes, each 80% Correct Non-Hodgkin lymphoma
2 to 3 cm in diameter. There is no evidence of lymphadenopathy in other
locations. 10% Wrong Tuberculosis
2% Other Ehrlichiosis
2% Other Listeriosis
The most likely cause of night sweats, weight loss, and cervical
lym phadenopathy in a patient with common variable immunodeficiency
receiving immunoglobulin treatment is non-Hodgkin lymphoma.
DETAILED FEEDBACK:
CLOSE
Copyright © 2022 Massachusetts Medical Society. All rights reserved
G) You are in Demo mode. Results are not saved.
A 44-year-old man returns for follow-up of abnormal ities that were noted on Which one of the following management approaches is the most
laboratory testing. He generally feels well and takes no medications. appropriate next step for this patient?
Laboratory testing, including a complete blood count, had been obtained for
an insurance evaluat ion: Not there yet...
CLOSE
Copyright © 2022 Massachusetts Medical Society. All nghts reserved
(D You are in Demo mode. Results are not saved.
An 82-year-old man reports progressive shortness of breath and 2.3 kg of Which one of the following diagnoses Is most likely In this case?
weight gain during the past several months. He was previously in good health,
except for primary hypertension and gastroesophageal reflux disease. Not there yet . II
On examination, he has a heart rate of 110 beats per minute and a blood
pressure of 1 02/64 mm Hg. He has significant purpura around both eyes. His 46% Correct Amyloid light-chain amyloidosis
j ugular venous pressure is elevated to the angle of his mandible. His heart
rhythm is tachycardic and regular The bases of both lungs have crackles. 15% Wrong Amyloid A amyloidosis
Both lower extremit ies exhibit edema.
Laboratory testing yields the following results: 21% Other Monoclonal gammopathy of undetermined significance
Creatinine (mg/dL) 1.8 0.8-1 .3 The most likely diagnosis in a patient wit h a monoclonal gam mopathy,
heart failure, renal fa ilure, and periorbital purpura is amyloid light-chain
An electrocardiogram shows reduced voltages in all leads. Serum protein amyloidosis.
electrophoresis shows an immunoglobulin G lambda M-protein level of 1.8
g/dL. Bone-marrow biopsy reveals 5% clonal plasma cells.
DETAILED FEEDBACK:
CLOSE
Copynght © 2022 Massachusetts Medical Society_ All nghts reserved
(D You are in Demo mode. Results are not saved.
A 60-year-old man with a 4-year history of low-grade B-cell lymphoma What Is the most likely d1agnos1s In this case?
presents to the emergency department with severe shortness of breath.
Two days ago, the patient presented to his primary care provider with a raised, Not there yet . II
purpuric rash on his legs and dorsal hands bilaterally, as well as edema and
paresthesias. During the past year, he has had arthralgia and myalgia with 62% Correct Cryoglobuli nemic vasculitis
increasing frequency.
In the emergency department, he is afebrile. His heart rate is 112 beats per 17% Wrong Granulomatosis with polyangiitis
minute, his blood pressure is 190/95 mm Hg, and his oxygen saturation is
92% while he is breathing ambient air. He has bilateral crackles in the lung 14% Other Polyarterit is nodosa
bases. The heart sounds are normal. He has no hepatomegaly or abdominal
masses. His lower extremities show 1+ pitting edema. He has raised purpura 10% Other Cold-agglutinin disease
on his hands and lower extremities bilaterally. He has no swelling of the wrists
or of the metacarpophalangeal or proximal interphalangeal joints. 3% Other Rheumatoid arthritis
Other than tachycardia, electrocardiography is unremarkable.
Laboratory findings are as follows: KEY LEARNING POINT:
CLOSE
Copynght © 2022 Massachusetts Medical Society. All rights reserved
(D You are in Demo mode. Results are not saved.
A 73-year-old man reports a 6-m onth history of recurrent low-grade fevers, a Which one of the fo llowing diagnoses is the most likely cause of the
3-month history of abdominal fullness, and a 6-week history of fatigue and anemia noted in this case?
moderately reduced exercise tolerance. He was previously in overall good
health. Not there yet..
On examination, he appears well and in no acute distress. His heart rate is 95
beats per minute, and his blood pressure is 132/64 mm Hg. He has several 25% Co rrect Autoimmune hemolyt ic anemia
palpable cervical and axillary lymph nodes (7 to 2 cm) that are nontender and
freely mobile. He has a palpable spleen t ip with inspiration. No hepatomegaly 4% Wrong Microangiopathic hemolytic anemia
or abdominal masses are present. The remainder of his physical examinat ion
is normal. Replacement of the bone marrow with chronic
58% Other
Laboratory results are as follows: lymphocytic leukemia
Neutrophil count (per mm3) 2100 1900-7600 An older patient presenting with a very elevated lymphocyte count and
anemia should be evaluated for chronic lymphocytic leukemia with
Lymphocyte count (per mm3) 29,200 800- 4100
autoimmune hemolyt ic anemia.
Lactate dehydrogenase (Li/liter) 352 100-190
The peripheral-blood smear reveals increased numbers of mature-appearing Chronic lymphocytic leukemia (CLL) occurs more frequently in older adults
lymphocytes, increased numbers of spherocytes, and polychromasia. than in younger individuals. Patients can present with an asymptomatic
lymphocytosis found incidentally on laboratory testing. Other features that
may be evident at presentation, or may develop later in the disease course,
include lymphadenopathy, splenomegaly, and systemic symptoms such as
fevers, night sweats, or weight loss.
Autoimmune hemolytic anemia occurs in 1 0% to 20% of pat ients with CLL
and does not depend on disease stage. In a patient with CLL, an elevated
lactate dehydrogenase level, reticulocytosis, and polychromasia (the term
for reticulocytosis as seen on the peripheral smear) suggest a diagnosis of
autoimmune hemolyt ic anem ia. The presence of anemia based on
autoimmune hemolysis in CLL does not convey the poorer prognosis
CLOSE
Copyrigh t © 2022 Massachusetts Medical Society. All nghts reserved.
(D You are in Demo mode. Results are not saved.
A 50-year-old woman with no significant medical history presents to the What Is the most likely underlying cause of this patient's presentation?
emergency department reporting 2 days of diffuse abdom inal pain without
nausea, vomit ing, or diarrhea. The pain is unrelated to eating meals. Not t here yet.. II
On examination, she has a temperature of 37. 1°C, a heart rate of 80 beats per
minute, and a blood pressure of 130/70 mm Hg. Her lungs are clear, and her 48% Correct Myeloproliferative neoplasm
heart rate and rhythm are regular. Her abdomen is moderately tender to
palpation in the epigast rium, with mild guarding but no rebound tenderness or 15% Wrong Factor V Leiden mutation
hepatosplenomegaly. She has no lower-extremity edema, and her neurologic
examinat ion is normal. 16% Other Paroxysmal nocturnal hemoglobinuria
Laboratory results are as follows:
9% Other Oral contraceptive use
Patient value Reference range
Leukocyte count (per m m 3) 9700 4500-1 1,000 12% Other Antiphospholipid syndrome
Platelet count (per mm 3) 375,000 150,000-350,000 The most common risk factor for Budd-Chiari syndrome is the presence of
Total bilirubin (mg/dL) 6.5 0.3- 1.0 a myeloproliferative neoplasm
CLOSE
Copyright © 2022 Massachusetts Medical Society_All rights reserved
(D You are in Demo mode. Results are not saved.
A 51-year-old man presents with progressive fatigue, early satiety, and fevers Which one of the following diagnoses Is most likely In this case?
without chills. He has a history of hepatit is C virus infection (genotype 3a)
diagnosed 5 years ago. After 3 months of antiviral therapy, he discontinued Not there yet .. II
treatment because of fatigue and mild depression. He has not had dental
work or any procedures within the past year. Review of systems is negative for
69% Correct Lymphoma
anorexia, nausea, vomiting, and diarrhea. He takes no medications except for
periodic ibuprofen.
9% Wrong HIV infection
He drinks one to t wo beers per week and has no history of heavy alcohol use.
He currently smokes cigarettes and has a 5-pack-year history. He does not 13% Other Epstein-Barr virus infection
use illicit drugs.
He weighs 70 kg, which is 3.6 kg lower than when measured one year ago. He 1% Other I Subacute bacterial endocarditis
does not appear ill but looks t ired . His general examination is unremarkable
other than a spleen that is palpable 9 cm below the left costal margin and 7% Other I Cirrhosis
several 2- to 5-cm lymph nodes in t he cervical, supraclavicular, axillary, and
inguinal regions bilaterally.
KEY LEARNING POINT:
Laboratory results are as follows:
- -
Patient value Reference range In a patient with hepatitis C virus infection, t he development of
splenomegaly and diffuse lymphadenopathy is most consistent wit h a
Leukocyte count (per mm3) 13,000 4500-11,000 diagnosis of low-grade lymphoma.
Lymphocytes(%) 80 22-44
Monocytes (%) 8 4-11 Hepatitis C virus (HCV) is associated with the development of lymphomas
of B-cell origin, particularly splenic lymphoma. The lymphoma often
Eosinophils (%) 2 0-8
regresses when the pat ient's HCV infection is treated.
Hemoglobin (g/dL) 13.0 13.5-17.5
Although this patient is at risk for cirrhosis, cirrhosis is not associated with
Mean corpuscular volume (µm 3) 88 80-100 lymphadenopathy.
Platelet count (per mm 3) 60,000 150,000-350,000 Epstein-Barr virus can cause lymphadenopathy, but it is most typically
associated with fever and pharyngitis. Thrombocytopenia is only rarely
Lactate dehydrogenase (U/liter) 180 100-190
associated with Epstein-Barr virus infection.
Aspartate aminotransferase (U/liter) 60 0-35 Endocarditis must always be considered in a patient with fever and
Alanine aminotransferase (U/liter) 65 0-35 splenomegaly, but subacute bacterial endocarditis would not cause diffuse
lymphadenopathy.
Albumi n (g/dL) 3.8 3.5-5.5
HIV infection more typically causes lymphopenia and anemia;
Prothrombin time (sec) 12.0 11.1-13.1
lymphocytosis is an uncommon finding with acute HIV infection HIV
CLOSE
Copyright © 2022 Massachusetts Medical Society_ All nghts rese1Ved
(D You are in Demo mode. Results are not saved.
A 70-year-old woman who has responded well to 6 months of treatment with What Is the most likely cause of this patient's abnormal hemogram?
oral methotrexate for symptomat ic rheumatoid arthritis is found to have an
abnormal complete blood count and other abnormal laboratory findings. She Not there yet . II
states that she does not experience dyspnea, fatigue, or neuropathy. She
drinks approximately one serving of alcohol per week.
71% Correct Methotrexate treatment
Her vital signs and physical examination are unremarkable.
Laboratory results are as follows: 4% Wrong Myelodysplastic syndrome
Leukocyte count (per mm3) 10,000 4500-1 1,000 15% Other Chronic inflammation
Platelet count (per mm 3) 180,000 150,000-350,000
Transferrin saturation(%) 16 15-45 This patient's macrocytosis is most likely due to treatm ent with
methotrexate, which inhibits dihydrofolate reductase. Methotrexate and
other drugs that inhibit DNA synthesis can cause macrocytosis.
Iron-deficiency anemia is characterized by microcytosis rather than
macrocytosis. Furthermore, the pat ient's iron studies do not suggest iron
deficiency. In iron deficiency, there is usually a low ferrit in level and an
increased total iron-binding capacity (TIBC). In addition, the transferrin
saturation (calculated as the serum iron level divided by the serum TIBC
level) is usually less than 10%; in this patient, it is 16%. The low iron level
and low TIBC in this case are likely secondary to the effects of chronic
inflammation, especially considering the patient's history of rheumatoid
arthritis and elevated ferritin. Anemia of chronic inflammation, however,
does not typically cause macrocytosis.
Myelodysplastic syndrome would typically cause m acrocytosis and can be
difficult to distinguish from methotrexate-induced macrocytosis. However,
it L nli~ •Iv n this scen1rio b1 'C'" Jse 'he oth, r line;:iqes (leu~ OC'' ·tes 'lnd
CLOSE
Copynght © 2022 Massachusetts Medical Society_ All rights reserved
(D You are in Demo mode. Results are not saved.
A 45-year-old woman presents with fatigue. One year ago, she underwent Which one of the fo llowing diagnoses Is most likely In this case?
Roux-en-Y gastric bypass surgery to treat morbid obesity and has since lost
more than 36 kg. She fol lows a regular diet and takes no medications Not there yet.. JI
Total iron-binding capacity (µg/dl) 500 250-370 Patients who have undergone gastric bypass surgery are prone to anemia.
In the setting of m icrocytic hypochromic anemia, t he likely diagnosis is iron-
A peripheral-blood smear shows hypochromic and microcytic erythrocytes. deficiency anemia, especially in menstruating women.
DETAILED FEEDBACK:
Patients who have undergone gastric bypass surgery are at risk for iron
deficiency and vitamin 8 12 deficiency, particularly if they have undergone a
malabsorptive procedure such as the Roux-en-Y gastric bypass. Iron
deficiency develops as food bypasses iron absorption sites in the
duodenum. Vitamin 8 12 deficiency develops because of inadequate bind ing
by intrinsic factor after gastric bypass.
Patients with iron-deficiency anemia are classically found to have
hypochromic, microcytic red cells on peripheral blood smear. Addit ional
laboratory testing will usually show a low iron level coupled with high iron-
binding capacity for a low iron saturation, low ferritin level, low mean
corpuscular volume (corresponding to the microcytosis seen on the
CLOSE
Copyright © 2022 Massachusetts Medical Society. All nghts reserved
(D You are in Demo mode. Results are not saved.
An 85-year-old woman reports a 4-month history of paresthesias in the tips of Which one of the fol lowing next steps in evaluation is most appropriate for
her fingers and toes, which she attributes to old age. She has a history of this patient?
osteoarthritis of the hands and knees (for which she takes acetaminophen as
needed) and heartburn (for which she takes omeprazole or famotidine You got itl
intermittently) but is otherwise in good health. Her weight has been stable for
many years, she eats a reasonably balanced diet, and she consumes no
Right
Measure serum methylmalonic acid and
alcohol. 74%
homocysteine levels
Her vital signs are normal, and examination of her head, neck, chest, and
abdomen is unremarkable. Neurologic examination identifies decreased 4% Other Check for the presence of anti parietal-cel l antibodies
vibratory and position sense in the feet, but sensation to pinprick and muscle
strength are intact. Check for the presence of anti-intrinsic factor
7% Other
Laboratory testing yields the following results: antibodies
Patient value Reference range 10% Other Obtain nerve conduction studies
Leukocyte count (per mm3) 9000 4500-11,000
5% Other Measure erythrocyte folate level
Hemoglobin (g/dl) 12.2 12.0-16.0
Mean corpuscular volume (µm 3) 97 80-100 In a patient with parasthesias, decreased vibratory and position sense in
the feet, a borderline-low vitamin B12 level, and a high-normal folate level,
Vitamin B12 (pg/ml) 260 >250
the most appropriate next test is measurement of serum methylmalonic
Folate, serum (ng/ml) 15.0 3.1-17.5 acid and homocysteine levels.
CLOSE
Copyright © 2022 Massachusetts Medical Society_ All nghts reserved
(D You are in Demo mode. Results are not saved.
A 68-year-old man presents with fatigue. He reports that for the past two Deficiency of which one of the following dietary components is this patient
years, he has taken several tablets of zinc daily to prevent colds. most likely to have?
His laboratory results are as follows:
Not there yet...
Patient value Reference range
4% Other Vitamin A
8% Other Iron
DETAILED FEEDBACK:
CLOSE
Copynght © 2022 Massachusetts Medical Society. All rights reseived
(D You are in Demo mode. Results are not saved.
Leukocyte count (per mm3) 3300 4500-1 1,000 The most likely cause of a sore tongue, macrocytosis, pancytopenia, and
paresthesias is vitamin B12 deficiency.
Platelet count (per mm 3) 88,000 150,000-3 50,000
CLOSE
Copynght © 2022 Massachusetts Medical Society. All rights reserved
(D You are in Demo mode. Results are not saved.
A 37-year-old woman with no significant medical history is admitted to t he What Is the most appropriate 1n1t1al therapy for this patient?
hospital with a 385°C fever and a mild petechial rash on her lower extremities.
On examinat ion, she is in no distress. Laboratory results are as fol lows: Not there yet JI
DETAILED FEEDBACK:
•
Thrombotic thrombocytopenic purpura (TTP) is a medical emergency
associated with a high mortality rate if left untreated. TTP arises in patients
-,·
mult imers, which activate platelets and cause ischemia related to platelet-
e· -
rich microthrombi.
--
Diagnosing TTP
0
,. ec, . • l
The diagnosis of TTP is based on the presence of microangiopathic
hemolysis (related to fragmentation of erythrocytes as they travel t hrough
damaged vessels) and thrombocytopenia (related to increased
consumption of platelets). Patients may also have neurologic symptoms,
0 0 renal fa ilure, and fever, but the classically described pentad is present in
-
<5% of patients with TTP
In this patient, hemolysis is confirmed by anemia accompanied by
CLOSE
Copyright © 2022 Massachusetts Medical Society_All rights reserved
(D You are in Demo mode. Results are not saved.
A 50-year-old man with end-stage kidney disease caused by polycystic kidney Which one of the following conditions is the most likely cause of this
disease who has received hemodialysis three t imes weekly for the past 6 patient's persistent anemia despite his receiving erythropoietin?
months now presents with progressive fat igue that does not improve with
dialysis. His medications include erythropoiet in 70,000 units subcutaneously Not there yet...
three times a week and oral ferrous sulfate 325 mg three t imes daily with
vitamin C. He reports no melena or rectal bleeding.
76% Correct Iron deficiency
His vital signs are normal, and a physical examination is unremarkable other
than the presence of a functioning fistula in the left forearm. Stool color is 3% Wrong Aluminum toxicity
brown. Three home tests for fecal occult blood have been negative.
His hemoglobin level has fa llen from 11.8 g/ dl (reference range, 13.5-17.5) 9% Other Myelodysplast ic syndrome
six mont hs ago to 10.2 g/ dl two months ago to 8.8 g/dl today. Other
laboratory results are as follows: 4% Other Occult gast rointestinal bleeding
DETAILED FEEDBACK:
Iron deficiency is the most likely cause of this patient's persistent anemia.
Iron deficiency is common in dialysis patients because of frequent blood
testing and blood lost from the procedure. Hemodialysis pat ients may have
absolute iron deficiency or functional iron deficiency (an inability to mobilize
stored iron adequately for erythropoiesis). Functional iron deficiency is
considered to be likely in pat ients with chronic kidney failure if they have a
ferritin level of :,:500 ng/ml or a transferrin saturation of :,:30%.
Iron supplem entation should be continued in hemodialysis patients with
ferrit in values up to 500 ng/ ml , as functional iron deficiency can occur in
pat ients with ferritin levels in the 100 to 500 ng/ml range. In many patients,
oral iron supplementation is inadequate, and intravenous supplementation
is required to opt imize the response to erythropoiet in-st imulating agents.
Inadequate erythropoiet 1 rPpl:irPrnent s L nlikelv in tr s C'SSP t Pr1usP the
CLOSE
Copyright © 2022 Massachusetts Medical Society. All nghts reseived
(D You are in Demo mode. Results are not saved.
A previously healthy 50-year-old woman reports progressive fatigue over the Which one of the following steps is most likely to lead to a diagnosis for
past several months as well as recent onset of epistaxis and heavy menstrual this patient?
bleeding. She does not take any medications and denies any recent travel or
infection. She drinks one glass of wine each night with dinner. Her family Not there yet...
history is negative for blood disorders.
Her vital signs and physical examination are remarkable only for pallor and a 75% Correct Refer for bone-marrow biopsy
resting heart rate of 92 beats per minute.
A complete blood count yields t he following results 4% Wrong Obtain an ultrasound of the liver and spleen
Bands(%) 0 0
Hemoglobin (g/dL) 8.5 12.0-16.0 Pancytopenia without peripheral blood smear evidence of dysplasia or
circulat ing blasts in an otherwise healthy individual is suggestive of aplastic
Mean corpuscular volume (µm 3) 101 80-100
anemia, which can be confirmed with bone-marrow biopsy.
Platelet count (cells/mm 3) 18,000 150,000-3 50,000
Vitamin B12 level, prothrombin t ime, partial t hromboplast in time, and The findings of pancytopenia with marked leukopenia and reticulocytopenia
fibrinogen level are all within normal limits. in this patient point toward a bone-marrow failure syndrome, such as
aplastic anemia, myelodysplastic syndrome, or paroxysmal nocturnal
No dysplasia or blasts are noted on peripheral blood smear.
hemoglobinuria. The absence of blasts or dysplasia findings on the
peripheral blood smear raises the possibility of aplastic anemia, requiring
prompt referral and confirm ation by bone-marrow biopsy.
Acquired aplastic anemia is a rare condition that affects previously healthy
individuals and leads to progressive cytopenias and bone-marrow failure
due to an immune response against hematopoietic stem cells. The mean
corpuscular volume is commonly elevated at presentation (i.e.,
macrocytosis) but may also be normal. Typical bone-marrow findings
include low overall cellularity on the core biopsy, absence of dysplasia, and
CLOSE
Copyright © 2022 Massachusetts Medical Society. All nghts reserved
(D You are in Demo mode. Results are not saved.
A 42-year-old man is hospitalized for progressive lethargy and intermittent Which one of the fol lowing diagnoses Is most li kely In th is patient?
confusion that has been worsening for the past week.
His medical history is significant for acute cholecystit is 6 months ago, for Not t here yet. . 11
CLOSE
Copyright© 2022 Massachusetts Medical Society. All rights reserved
(D You are in Demo mode. Results are not saved.
A 35-year-old man who has been a vegetarian for the past 5 years presents Which one of the following laboratory tests should be ordered next to
for his annual physical examination. He is an avid runner and is currently evaluate this patient's anemia?
training for a marathon.
On physical examination, his vital signs are normal, and his BMI is 27 .2. He Not there yet...
takes no medications or supplements. An in-office point-of-care hemoglobin
level is 10.5 g/dl (reference range, 13.5-17.5). 69% Correct Complete blood count wit h red-cell indices
DETAILED FEEDBACK:
Iron deficiency is less common in men than in women, but this patient is at
risk because of his vegetarian diet and participation in an endurance sport
Laboratory evidence of iron deficiency occurs in stages, with initial
preservation of red-cell morphology and count despite a decrease in iron
stores. As iron depletion cont inues, the red-cell distribution width increases,
ferrit in levels are m easurably low, and the classic picture of iron-deficiency
anemia with hypochromia and microcytosis appears.
When a screening hemoglobin level is found to be abnormally low, as in this
case, the next step is a complete blood count with red -cell indices.
Hemoglobin elect rophoresis would be indicated if an underlying inherited
hemoglobinopathy, such as sickle cell disease or a thalassemia, was
suspected. It is not the appropriate initial step in evaluat ing a low screening
hemoglobin level.
CLOSE
Copynght © 2022 Massachusetts Medical Society_ All nghts reserved
(D You are in Demo mode. Results are not saved.
A 27-year-old woman presents for her fi rst med ical visit after immigrating to Which one of the following tests is most likely to confirm the suspected
the United States. She has generally been healthy and takes no medications. diagnosis in this patient?
She reached menarche at age 13 and has normal menses, with moderate flow
that lasts 5 days. Not t here yet. ..
Her blood pressure is 110/70 mm Hg, and her heart rate is 95 beats per
minute. Her physical examination is normal. 80% Correct Hemoglobin electrophoresis
Laboratory analysis is performed as part of screening for a new immigrant. All
results are normal with the exception of some elements of the complete blood 2% Wrong Haptoglobin testing
count:
12% Other Measurement of ferritin
Pat ient value Reference range
Leukocyte count (per m m 3) 9000 4500-1 1,000 3% Other Measurement of vitamin B12
CLOSE
Copyright © 2022 Massachusetts Medical Society. All rights reserved
(D You are in Demo mode. Results are not saved.
A 69-year-old woman with Alzheimer disease who resides in a nursing home Which one of the following additional laboratory tests should this pat ient
is referred for evaluation of a new anemia. The staff noticed that she has been undergo next?
more lethargic and less interactive during the past 3 months, and a flngerstick
test conducted there revealed a hemoglobin level of 7.1 g/ dl (reference range, Not there yet...
120- 160)
On examination, the patient appears pale. She is alert and oriented to name 87% Correct Serum methylmalonic acid
and place but is confused about t he year. Neurologic examination reveals a
shuffling, broad-based gait and diminished sensation to light touch and 4% Wrong Flow cytometry for COSS and CDS9 proteins
pinprick
Laboratory testing yields the following results: 3% Other Direct Coombs test
Reticulocyte count (per mm 3) 23,000 20,000-80,000 In a patient with a severe macrocytic anemia associated wit h neurologic
findings, an elevated serum methylmalonic acid level is very specific for
Haptoglobin (mg/dl) 30 16-199
identifying vitamin B12 deficiency
Lactate dehydrogenase (U/liter) 185 100-190
Urine testing for free hemoglobin and hemosiderin is negative. A peripheral This pat ient's peripheral blood smear shows many large, oval-shaped
blood smear is obtained (figure). erythrocytes (macrocytic anemia), and the single neutrophil has at least six
visible lobes (neutrophil hypersegmentation) Clinical evaluation of
macrocyt ic anemia begins wit h evaluation of t he most likely causes, such
as nutritional deficiencies (e.g., vitamin B12 deficiency), excess alcohol use,
myelodysplastic syndrome, and (more rarely) severe hypothyroidism.
Vitamin B12 deficiency has many causes, including diseases of the ileum
(e g, inflammatory bowel conditions such as Crohn disease), gastrectomy,
pernicious anemia (related to autoantibodies against intrinsic factor),
excess alcohol use, and lack of adequate dietary intake (e.g., because of
adherence to a strict vegan diet). Mild or subclinical manifestations of
vitamin B12 deficiency are common in older adults and may stem from one
or more of the aforementioned factors, as well as from other factors that
CLOSE
Copynght © 2022 Massachusetts Medical Society. All rights reserved
(D You are in Demo mode. Results are not saved.
A 75-year-old woman present s for a health maintenance visit She reports Which one of the following tests should be ordered to establish a diagnosis
progressive fat igue and shortness of breath on exert ion. She has also had in this patient?
persistent hip pain for t he past 6 months that is partially relieved by daily
naproxen. Not there yet...
She has a heart rate of 90 beats per minute, a blood pressure of 124/64 mm
Hg, and an oxygen saturation of 95% while breathing ambient air. She appears 74% Correct Iron studies
pale and has clear lungs and a regular heart rate without murmur.
Examination of her extremities reveals right-knee crepitus and diminished 2% Wrong Measurement of homocysteine
range of motion of the right hip.
A complete blood count (CBC) was normal one year ago. Today's CBC results 13% Other Bone-marrow biopsy
are as follows:
8% Other Hemoglobin electrophoresis
Patient value Reference range
Leukocyte count (per mm3) 5500 4500-11,000 2% Other Measurement of vitamin B12
Platelet count (per mm 3) 350,000 150,000-350,000 The most common cause of m icrocytic anem ia is iron deficiency.
Reticulocytes (% red cells) 1.0 0.5-2.5
CLOSE
Copyright© 2022 Massachusetts Medical Society. All rights reserved
(D You are in Demo mode. Results are not saved.
A 69-year-old woman presents with fatigue and reduced exercise tolerance. Which one of the fol lowing diagnoses Is most likely In this case?
One month ago, she had a cough and fever and was treated for an atypical
community-acquired pneumonia with azithromycin. She has overweight but Not there yet... 11
Platelet count (per mm 3) 174,000 150,000-350,000 The most likely diagnosis in a patient who presents after an atypical lung
infection with hemolytic anemia and a Coombs test that is positive for anti-
Mean corpuscular volume (µm 3) 102 80-100 C3 and negative for anti-immunoglobulin G is cold agglutinin hemolytic
Lactate dehydrogenase (U/liter) 315 100- 190 anemia.
CLOSE
Copyright © 2022 Massachusetts Medical Society_All nghts reserved
(D You are in Demo mode. Results are not saved.
A 52-year-old woman is evaluated for fatigue and a 3-week history of dyspnea Which one of the fol lowing diagnostic tests is most appropriate for this
on exertion that has made her job progressively more difficult. She has no patient?
personal or family history of anemia, and she takes no medications.
On examination, the patient appears pale, and her eyes are icteric. She has a You got it!
heart rate of 106 beats per minute, a respiratory rate of 16 breaths per m inute,
and a blood pressure of 113/82 mm Hg. She has no appreciable adenopathy. 57% Right Direct antiglobulin test
The tip of her spleen is palpable 2 cm below t he costal margin.
Laboratory testing yields the following results: 6% Other Indirect antiglobulin test
Creat inine (mg/dL) 1.0 0.6- 1.1 The diagnostic test of choice for autoimmune hemolytic anemia is a direct
antiglobulin test.
Lactate dehydrogenase (U/liter) 425 100- 190
A peripheral-blood smear shows normocyt ic eryt hrocytes with 2+ Evidence of hemolysis and a peripheral spherocytosis suggests
spherocytes. Polychromasia is present. Leukocyte and platelet morphology autoimmune hemolytic anemia. The confirmatory test for this condit ion is a
are normal. direct antiglobulin test.
An indirect antiglobuli n test, which detects preformed antibodies in the
plasma that are not attached to erythrocytes, is used for screening before a
blood transfusion.
Osmotic fragility is an assay for hereditary spherocytosis, a diagnosis that
is unlikely in the absence of fam ily or personal history.
A glucose-6-phosphate dehydrogenase (G6PD) assay is appropriate to
detect G6PD deficiency, which can lead to hemolysis after exposure to fava
beans, infections, or certain oxidant medications. The characteristic cell on
peripheral-blood smear is a bite cell. G6PD maps to the X chromosome.
CLOSE
Copynght © 2022 Massachusetts Medical Society_ All rights reserved
(D You are in Demo mode. Results are not saved.
A 50-year-old man reports 2 weeks of worsening fatigue. He has a history of Which one of the following initial treatments is most appropriate for this
difficult-to-control ulcerative colitis, but all immunosuppressive therapy was pat ient?
discontinued after a total colectomy and end ileostomy one month ago. He
has not had any blood in his ileostomy bag since hospital discharge. Not there yet...
He has normal vital signs but appears mi ldly jaundiced and fatigued. His
abdominal scars are wel l healed, and he has no abdominal pain or distention. 65% Correct Prednisone with or w ithout rituximab
His spleen tip is palpable 2 cm below the costal margin.
Laboratory testing yields the following results: 3% Wrong Azathioprine
CLOSE
Copyright © 2022 Massachusetts Medical Society_All nghts reserved
(D You are in Demo mode. Results are not saved.
A 65-year-old man with a history of type 2 diabetes and end-stage kidney Which one of the following management strategies is the most appropriate
disease presents with diarrhea, nausea, emesis, and malaise after missing his next step for this patient?
dialysis appointments for a week. He is started on sustained low-efficiency
dialysis. Not there yet...
On examination, he is afebrile and has a heart rate of 76 beats per minute and
a blood pressure of 702/64 mm Hg. His heart rhythm is regular, and a 72% Correct Start an erythropoiesis-stimulat ing agent
midsystolic ejection murmur is appreciated. His face and conjunctivae are
pale. 6% Wrong Transfuse packed red cells
Laboratory testing yields the followi ng results:
- 13% Other Start intravenous iron
Patient value Reference range
Leukocyte count (per mm3) 4500 4500-1 1,000 7% Other Initiate treatment with folate and vitamin C
CLOSE
Copynght © 2022 Massachusetts Medical Society. All nghts reserved
(D You are in Demo mode. Results are not saved.
A 50-year-old woman with a long-standing history of rheumatoid arthritis and What Is the most likely cause of this patient's anemia?
fatigue is found to be anemic. She takes hydroxychloroquine and intermittent
glucocorticoids to control her rheumatoid arthritis. There have been no recent Not there yet . 11
DETAILED FEEDBACK:
CLOSE
Copyright © 2022 Massachusetts Medical Society. All nghts reserved
G) You are in Demo mode. Results are not saved.
A 72-year-old woman presents with new numbness and tingling in both feet. Which one of the following is the most likely cause of this patient's
Her history includes long-standing type 2 diabetes (well controlled on symptoms?
metformin 1000 mg t wice daily), hypertension (treated with lisinopril), and
gastroesophageal reflux disease (treated with omeprazole 40 mg daily). She Not there yet..
does not have a previous history of neuropathy. Her review of systems is
otherwise negative.
67% Correct Medicat ion-induced vitamin deficiency
Her heart rate is 72 beats per minute, and her blood pressure is 110/64 mm
Hg. She has diminished vibratory and position sense in both feet and mild 20% Wrong Inadequate dietary vitamin intake
hyperreflexia on test ing of ankle reflexes.
Laboratory testing yields the following results: 4% Other Medicat ion-induced hemolytic anemia
Reticulocyte count(% red cells) 1.0 0.5-2.5 Long-term use of metformin or a proton pump inhibitor has been
associated with development of low levels of vitamin B12.
Creatinine (mg/d L) 0.8 0.6-1 .1
DETAILED FEEDBACK:
CLOSE
Copyright © 2022 Massachusetts Medical Society. All nghts reserved
(D You are in Demo mode. Results are not saved.
A 50-year-old man with a history of inflammatory bowel disease has a follow- Which one of the following treat ments is most appropriate for treating this
up appointment to discuss his anemia, which is from ongoing gastrointestinal patient's anemia?
blood loss related to his inflammatory bowel disease.
When the anemia was diagnosed 3 months ago, the patient was started on Not there yet...
ferrous sulfate 325 mg once daily by mouth. Despite this supplement, he has
persistent fatigue and exercise intolerance. In addition, he reports nausea and 85% Correct Int ravenous iron
abdominal pain after taking the iron. He is ot herwise doing well and is in no
distress. His vital signs and physical examination are normal, other than pallor 3% Wrong Packed red-cell transfusion
and minimal diffuse tenderness over the abdomen.
Laboratory findings are as follows: 6% Other Oral iron and vitamin C
Ferritin (ng/m L) 5 30-300 The most appropriate treatment for a patient with moderate iron-deficiency
anemia who cannot tolerate or does not respond to oral iron replacement is
Iron (µg/dL) 70 30-760
intravenous iron.
Transferrin (mg/dL) 500 230-390
CLOSE
Copynght © 2022 Massachusetts Medical Society. All rights reserved
(D You are in Demo mode. Results are not saved.
A 35-year-old woman reports a 2-month history of progressive fatigue, Which one of the fol lowing assessments is most likely to reveal the cause
weakness, diminished exercise capacity, a 2.3-kg weight gain, and heavy, of this patient's symptoms?
irregular menses. She was previously in overall good health.
On examination, her heart rate is 56 beats per m inute, and her blood pressure Not t here yet...
is 109/65 mm Hg. She moves and speaks slowly. Her oropharynx and sclera
are unremarkable. She has mild conjunctiva! pallor. Her jugular venous 76% Correct Thyroid-stimulating hormone level
pressure is normal. She has no lymphadenopat hy. Her heart rhythm is regular.
Her lungs are clear bilatera lly. Her extremities show no edema or cyanosis. 10% Wrong Serum ferritin levels
Her laboratory results are as follows:
2% Other Erythrocyte sedimentation rate
Patient value Reference range
CLOSE
Copynght © 2022 Massachusetts Medical Society. All rights reserved
(D You are in Demo mode. Results are not saved.
An 80-year-old man with fatigue, chronic kidney disease, and well-controlled Which one of the following treat ments Is 1nd1cated for this patient?
hypertension is evaluated for fo llow-up of anemia. His physical examination is
normal. Not there yet .. 11
DETAILED FEEDBACK:
CLOSE
Copynght © 2022 Massachusetts Medical Society. All nghts reserved
(D You are in Demo mode. Results are not saved.
A 47-year-old woman with a history of systemic lupus erythematosus reports Which one of the following laboratory indicators would confirm the cause
3 days of fat igue, rash, and muscle pain. She is not currently taking any of this patient's anemia?
medications.
Vital signs are within normal limits, with the exception of a heart rate of 105 You got it!
beats per minute. She has a faint malar rash and an erythematous
maculopapular rash on bot h lower extremities. 70% Right Direct antiglobulin test
Laboratory testing yields the following results
16% Other Haptoglobin level
Patient value Reference range
Leukocyte count (per mm3) 6500 4500-11 ,000 3% Other Total bil irubin level
DETAILED FEEDBACK:
CLOSE
Copynght © 2022 Massachusetts Medical Society_ All nghts reserved
(D You are in Demo mode. Results are not saved.
A 33-year-old previously healthy man reports t hat he has had diarrhea and Which one of the following diagnoses Is most likely In this patient?
fatigue for the past several days . His wife notes that he has "not been act ing
like himself" during t he past week. He does not take any medications. Not there yet . II
On physical examination, the patient is afebrile. He has a blood pressure of
140/1 00 mm Hg and a heart rate of 110 beats per minute. He is alert and
86% Correct
I Thrombot ic thrombocytopenic purpura/hemolytic-
oriented to name and place, but not to year. uremic syndrome
His abdomen is diffusely tender to palpation, without rebound or guarding.
Retinal examination is unremarkable. He is jaundiced and has several areas of 2% Wrong Cancer involving the bone marrow
petechiae on his lower extremities.
4% Other Disseminated intravascular coagulation
Laboratory test ing shows the following results
CLOSE
Copyright © 2022 Massachusetts Medical Society_All rights reserved
(D You are in Demo mode. Results are not saved.
A 48-year-old man presents with recurrent episodes of epistaxis that are What Is the most likely d1agnos1s In this patient?
intermittently severe. During the past year, he has had m igraine headaches at
least once a month. He also reports fatigue with dyspnea on exertion during Not there yet . I
the past several months. His father died at age 51 from a ruptured brain
aneurysm.
73% Correct Hereditary hemorrhagic telangiectasia
The patient's temperature is 37.5°C, his heart rate is 70 beats per minute, and
his blood pressure is 105/70 mm Hg. Small red spots that blanche with mild 14% Wrong Peutz-Jeghers syndrome
pressure are noted on the buccal mucosa, tongue, and lips. He has subtle
clubbing and cyanosis of the fingers. The remainder of the physical 3% Other Lymphoplasmacytic lymphoma
examination is normal.
A neurologic examination is normal. Chest radiography is notable for a round, 2% Other Eisenmenger syndrome
well-circumscribed mass, 2 cm in diameter, in the left lower lobe.
8% Other Granulomatosis with polyangiitis
Laboratory evaluation reveals the following:
Mean corpuscular volume (µm 3) 70 80-100 The most likely diagnosis in a patient with recurrent epistaxis,
telangiectasia, and anemia is hereditary hemorrhagic telangiectasia.
Hemoglobin (mg/dL) 84 13.5- 17.5
CLOSE
Copyright © 2022 Massachusetts Medical Society. All rights reserved
(D You are in Demo mode. Results are not saved.
A 34-year-old woman presents to the emergency department with a 3-day Which one of the following tests is most appropriate for establishing a
history of severe abdominal pain, nausea, vomiting, and constipation. She also diagnosis in this patient?
describes cold sweats, weakness, and confusion. Five days ago, she
presented to her primary care provider complaining of dysuria; her urinalysis Not there yet...
was positive for nitrites and leukocyte esterase. She was prescribed a course
of trimethoprim-sulfamethoxazole for uncomplicated cyst itis. Her current
80% Correct Urine screen for porphobilinogen
symptoms started 2 days after initiating the antibiotic, and her urine became
red-tinged.
4% Wrong Plasma metanephrine screening
The patient was previously healthy but has had several less-severe episodes
of similar abdominal pain during the past 10 years The cause of the recurrent 5% Other Antineutrophil cytoplasm ic antibody testing
abdominal pain is unknown. She current ly takes oral contraceptive pills.
On examination, her temperature is 37.4°C, her heart rate is 105 beats per 3% Other Cerebral spinal fluid analysis
minute, and her blood pressure is 155/ 90 mm Hg. She is confused, restless,
and diaphoretic. Her breaths are shallow. She has mild, diffuse abdominal 9% Other Urine test for eosinophils
tenderness but no rebound or flank tenderness. No skin lesions are apparent
She has reduced muscle strength throughout her arms and legs.
KEY LEARNING POINT:
Initial laboratory testing yields the following results:
Patient value Reference range The initial diagnostic study of choice for acute porphyria is a rapid urine
screen for porphobilinogen.
Leukocyte count (per mm3) 9000 (with normal 4500- 11,000
differential)
DETAILED FEEDBACK:
Hemoglobin (g/d L) 13.1 12.0-16.0
The classic signs and symptoms of an acute porphyria attack include
Platelet count (per mm 3) 350,000 750,000-350,000 severe abdominal pain with nausea, vomiting, and constipation; both
Sodium (mEq/ liter) 723 736-745 sensory and peripheral neuropathies with numbness and muscle
weakness; autonomic nervous system dysregulation and increased
Serum creatinine (mg/ d L) 0.7 0.6-1 .1
catecholami nes with diaphoresis, neuropsychiatric abnormalities,
Urinalysis is negative for nitrites, leukocyte esterase, red cells, and erythrocyte hypertension, and tachycardia; hyponat remia; and red-tinged urine (due to
casts. A noncontrast CT scan of t he abdomen and pelvis is normal. the presence of porphyrins). Sulfonamides (such as trimethoprim-
sulfamethoxazole), barbiturates, and hydantoins (such as phenytoin) can
precipitate attacks in pat ients with acute porphyria. Test ing for increased
porphobilinogen in a spot urine sample is fast and sensit ive for diagnosing
acute porphyria, especially at the time of an acute attack.
Guillain - Barre syndrome (GBS) classically manifests with acute,
progressive, symmetric muscle weakness. Analysis of cerebrospinal fl uid
for an albumin-cytological dissociation is part of the workup for GBS
CLOSE
Copynght © 2022 Massachusetts Medical Society. All rights reserved
(D You are in Demo mode. Results are not saved.
Total bilirubin (mg/d L) 0.9 0.3-1.0 Gaucher disease, the most common lysosomal storage disease, is an
autosoma l recessive disorder caused by a deficiency of the
Direct bilirubin (mg/d i) 0.2 0.1 -0.3
glucocerebrosidase enzyme, causing t he accumulation of glycolipids.
Aspartate aminotransferase (U/liter) 55 0-35 Clinical manifestat ions of the disease include hepatosplenomegaly and
bone disease. Bone disease may manifest as bone pain or swelling, aseptic
Alanine aminotransferase (U/liter) 75 0-35
necrosis, or osteoporosis. This patient's imaging reveals flaring of t he distal
Al kal ine phosphatase (U/liter) 361 30- 120 femur, the most typical finding; there is also evidence of marrow infarctions,
and there are secondary osteoarthritic changes due to subchondral bony
The differential count is normal; the peripheral-blood smear is normal other collapse resulting from osteonecrosis. Splenomegaly can lead to
than the finding of pancytopenia. There is no myeloid immaturity or blasts. thrombocytopenia and anemia from splenic sequestration. Type 1 Gaucher
There are no teardrop-shaped or nucleated red cells. There is no obvious disease, the most common type, rarely causes neurologic symptoms or
dysplasia on the smear. signs and is more common among Ashkenazi Jews.
A dual-energy x-ray absorptiometry scan, performed to further evaluate t he Chronic myeloid leukemia would be unlikely, given a lack of myeloid
pain in her hips and knees, reveals moderate osteoporosis (figure). immaturity and the absence of an elevated leukocyte count
CLOSE
Copyright © 2022 Massachusetts Medical Society. All rights reserved
(D You are in Demo mode. Results are not saved.
A 30-year-old man presents durin g the summer with new onset of bl istering Which one of the following diagnoses Is most likely In this case?
lesions and bullae on his hands and other sun-exposed areas . He smokes
cigarettes daily, consumes six servings of alcohol daily, and has a distant Not there yet. II
history of intravenous drug use. He denies abdominal pain.
Vital signs are unremarkable. Physical examinat ion reveals periorbital hair Correct
Porphyria cutanea tarda
growth and hyperpigmentation of his hands, forearms, face, and neck. Answer
DETAILED FEEDBACK:
CLOSE
Copyright © 2022 Massachuseus Medical Society. All nghts reserved
(D You are in Demo mode. Results are not saved.
A 24-year-old woman with systemic lupus erythematosus reports increased Which one of the following diagnoses 1s most likely in this patient?
bleeding. For the past several months, her periods have been heavier than
usual, and she has been bruising easily. She also notes that her gums Not there yet .. 11
Other
Answer
Lupus anticoagulant
DETAILED FEEDBACK:
CLOSE
Copynght © 2022 Massachusetts Medical Society_ All nghts reserved
(D You are in Demo mode. Results are not saved.
An 18-year-old female cross-country runner reports fatigue and a decline in Which one of the following treatment recommendations Is most
her running performance during the past 3 months. She reports no difficulty appropriate for this patient?
sleeping or unusual stressors in her life. She is a vegetari an but states that
she eats a healthy, well-balanced diet and does not have any restrictive eating
habits. Her menstrual periods are regular and light.
Laboratory findings are as follows: Your
Answer
Take a standard oral elemental iron supplement
Pat ient value Reference range
Other
Leukocyte count (per mm3) 7600 4500-11,000 Take an iron-containing multivitamin
Answer
Erythrocyte count (per mm 3) 4,030,000 4,000,000-5,200,000
Other
Hemog lobin (g/ dL) 12.3 12.0-16.0 Answer
Include several iron-rich vegetables in the daily diet
Red-cell distri bution width(%) 13.9 11 .5- 14.5 KEY LEARNING POINT:
Platelet count (per mm 3) 176,000 150,000-350,000 The most appropriate treatment for iron deficiency without anemia in a
Ferritin (ng/mL) 12 10- 200 female athlete is daily elemental iron.
DETAILED FEEDBACK:
Iron deficiency without anemia (i.e., a low ferritin level with a normal
hemoglobin level) is common in com petitive athletes, especially women.
Treatment with iron supplementation is generally recommended for
athletes who have a ferritin level below 30 ng/ m L (reference range,
10-200), even if the hemoglobin level is normal. The most appropriate
treatment in this setting is oral iron replacement.
There is considerable disagreement about the best regimen to replace iron
in this setting. Recent data have suggested t hat taking iron on alternating
days is associated with improved iron absorption in the short term, but the
long-term implicat ions of thi s, as well as the effect on adherence of a more
complex dosing regimen, remain unclear. Therefore, a reasonable approach
CLOSE
Copyright @ 2022 Massachusetts Medical Society. All righ1s reserved.
(D You are in Demo mode. Results are not saved.
A 61-year-old man is found to have a low platelet count on a routine complete Which one of the following diagnoses is most li kely In t his case?
blood count. The laboratory technician reports evidence of platelet clumping
on the peripheral-blood smear. Not t here yet II
The patient has a history of atrial fibrillation, hypertension, and hyperlipidemia.
He is treated with rivaroxaban, lisinopril, and simvastat in. He also takes Correct
Answer
Pseudoth rom bocytopen ia
quinine for intermittent leg cramps.
He reports no history of illicit drug use, sexual intercourse without con doms,
Your
or blood transfusions. There is no known history of a familial bleeding 0 Answer
Thrombotic thrombocytopenic purpura
disorder.
Other
His temperature is 37.3°C, his blood pressure is 130/85 m m Hg, and his heart Answer
Immune thrombocytopenia
rate is 84 beats per m inute. His examination is unremarkable, and
hepatosplenomegaly, ecchymoses, and petechiae are absent. Other
Answer
Disseminated intravascular coagulation
Laboratory results are as follows:
DETAILED FEEDBACK:
CLOSE
Copyright© 2022 Massachusetts Medical Society. All rights reserved
(D You are in Demo mode. Results are not saved.
A 65-year-old woman with congestive heart failure and type 1 von Willebrand Which one of the following management approaches is most appropriate
disease presents to the emergency department with an enlarging hematoma for th is patient?
on her right breast. Three days earlier, she underwent biopsy of a suspicious
right breast lesion found on mammogram. Thirty minutes before t he biopsy, You got itl
she was given an intravenous dose of desmopressin and had an appropriate
•
increase in her von Willebrand factor ristocetin cofactor activity level. The
Your
surgery was not immediately complicated by excessive bleed ing. Answer
Administration of von Willebrand factor concentrate
Platelet count (per mm3) 310,000 150,000- 350,000 KEY LEARNING POINT:
Sodium (mEq/liter) 128 136-145 A pat ient with von Wil lebrand disease who continues to bleed despite
prophylaxis with desmopressin should be treated with von Wil lebrand
factor concentrate.
DETAILED FEEDBACK:
CLOSE
Copyright © 2022 Massachusetts Medical Society. All rights reserved
(D You are in Demo mode. Results are not saved.
A 35-year-old woman presents to clinic fo r follow-up 7 days after initiating In addition to admitting the patient to the hospital and obtaining a vascular
enoxaparin 50 mg every 12 hours (7 mg/kg body weight) for a pulmonary surgery evaluation, which one of the following management strategies is
embolism that developed after a long airline flight. She has had significant most appropriate for this patient?
improvement in her dyspnea and chest pain. She has not noted any bleeding
or bruising but does report new pain, swelling, and discoloration in her left leg. Not there yet...
She is afebrile and has a heart rate of 715 beats per minute at rest, a blood
pressure of 102/76 mm Hg, and an oxygen saturation of 98% while breathing 61% Correct Stop enoxaparin, and start argatroban
ambient air. Her left leg is swollen and red below the knee. Dorsal is pedis
pulses are strong in the right foot but not palpable on the left. Her left great 73% Wrong Continue enoxaparin alone
toe is dusky in appearance and painful to the touch. The remainder of the
examination is normal. 72% Other Continue enoxaparin, and initiate warfarin
Laboratory testing reveals a platelet count of 43,000 per compared with mm 3,
183,000 per mm 3 one week ago (reference range, 150,000-350,000). Other 6% Other Stop enoxaparin, and initiate rivaroxaban
laboratory results are as follows:
Stop enoxaparin, and start intravenous unfractionated
Patient value Reference range 8% Other
heparin
Hemoglobin (g/dL) 14.2 12.0-76.0
DETAILED FEEDBACK:
CLOSE
Copyright © 2022 Massachusetts Medical Society_All nghts reserved
G) You are in Demo mode. Results are not saved.
A 30-year-old man presents for follow-up of immune thrombocytopenia, which Which one of the following management approaches is most appropriate
was diagnosed 5 years ago and treated with laparoscopic splenectomy 4 for this patient?
years ago. He has not had any bleed ing episodes and is not taking any
medications. Not there yet...
Physical examination is unremarkable except for well-healed surgical scars
related to t he splenectomy. There are no petechiae or ecchymoses. Correct Observe and repeat complete blood count in one
A complete blood count, taken 6 months ago, revealed a platelet count of Answer month
185,000 per mm 3 (reference range, 150,000-350,000).
Your Initiate prednisone 60 mg daily and repeat platelet
Laboratory testing yields the followi ng results: 0 Answer count in one week
Patient value Reference range
Other Measure international normalized ratio, partial-
Leukocyte count (per mm3) 4800 (normal differential) 4500-11,000
Answer
I thromboplastin time, D-dimer, and flbrinogen
Hemoglobin (g/dL) 14.0 13.5-17.5
Other Perform CT of the abdomen to look for accessory
Hematocrit (%) 42 41.0-53.0
Answer spleen
Platelet count (per mm 3) 56,000 150,000-350,000
Other
Perform bone-marrow examination with cytogenetics
A peripheral-blood smear confirms moderate thrombocytopenia; some red Answer
cells show Howell-Jolly bodies.
Blood chemistry results, including renal and liver function tests, are normal.
KEY LEARNING POINT:
DETAILED FEEDBACK:
CLOSE
Copyright © 2022 Massachusetts Medical Society_ All nghts reserved
(D You are in Demo mode. Results are not saved.
A 40-year-old man with a history of ankylosing spondylitis reports a 6-mont h Which one of the following tests is most likely to confirm the diagnosis in
history of intermittent diarrhea and abdominal pain. During this period, he lost this patient?
6.8 kg and had occasional low-grade fevers.
His vital signs and cardiorespiratory and neurologic examinations are normal. Not there yet...
The abdomen is diffusely tender in the left lower quadrant, with hyperactive
bowel sounds but without hepatosplenomegaly. No skin rash is noted, and the Correct
Colonoscopy
joints appear normal. Answer
Monocytes (%) 18 4-11 The typical causes of a reactive monocytosis are autoimmune conditions.
Absolute monocyte 1850 100-800
count (per mm3) DETAILED FEEDBACK:
The differential blood count is otherwise unremarkable. Liver function test Monocytosis can have secondary causes, such as chronic infections (e.g.,
results and lipase levels are normal. The monocyte count in a peripheral-blood tuberculosis, brucellosis, listeriosis), vasculitis or other autoimmune
smear is elevated, and the cells appear mature without obvious dysplasia. conditions (e.g., systemic lupus erythematosus, inflammatory bowel
disease), sarcoidosis, and cancers (e.g., Hodgkin disease). It also can have
primary causes - bone-marrow disorders such as acute monoblastic
leukemia and chronic myelomonocytic leukemia.
In this patient, who has a history of ankylosing spondylitis and recurrent
abdominal pain with diarrhea, weight loss, and low-grade fevers, an
inflammatory bowel disorder such as ulcerative colitis or Crohn disease is
an important possibility to consider. The test that is most likely to diagnose
an inflammatory bowel disorder that is causing a reactive monocytosis is a
colonoscopy to the ileum, with biopsies. If the colonoscopy is normal, a CT
CLOSE
Copyright @ 2022 Massachusetts Medical Society. All nghts reserved
(D You are in Demo mode. Results are not saved.
A 49-year-old woman with chronic renal insufficiency is hospitalized with In addition to discontinuing this patient's unfractionated heparin, which one
chest pain and diagnosed with non-ST-segment elevation myocardial of the following management approaches is most appropriate?
infarct ion. This illness has been managed with simvastatin, lisinopril,
metoprolol, and unfractionated heparin Her platelet count on admission is Not t here yet. ..
387,000 per mm 3 (reference range, 750,000-350,000).
Six days later, her platelet count fa lls to 43,000 per mm 3. She also develops 80% Correct Start argatroban
new swelling in her lower left extremity. There are no signs of bleeding.
Ultrasound examination of t he lower extremities confirms deep-vein 2% Wrong Transfuse platelets
thrombosis.
70% Other Start dalteparin
DETAILED FEEDBACK:
CLOSE
Copyright © 2022 Massachusetts Medical Society_ All rights reserved
(D You are in Demo mode. Results are not saved.
A healthy 35-year-old woman presents to discuss an abnormal complete Which one of the following diagnoses Is most likely In this case?
blood count ordered by her gynecologist.
Laboratory testing yields the following results: Not there yet .. II
Patient value Reference range
63% Correct Beta-thalassemia trait
Hemoglobin (g/dl) 11 .8 12.0-16.0
DETAILED FEEDBACK:
CLOSE
Copyright © 2022 Massachusetts Medical Society_All nghts rese1Ved
(D You are in Demo mode. Results are not saved.
A 73-year-old woman with a history of hypertension and chronic obstructive Which one of the following conditions is the most likely cause of this
pulmonary disease presents to the emergency department with red streaking patient's thrombocytopenia?
on her right arm. She grazed t he arm 2 days ago and stopped m inimal
bleeding from the wound by applying local pressure. She reports a fever of Not there yet. ..
38.7°C.
She has a platelet count of 243,000 per mm 3 (reference range, Correct
Drug-induced thrombocytopenia
150,000-350,000). She is started on intravenous vancomycin and ceftriaxone. Answer
She is then hospitalized, improves quickly, and is discharged on day 3.
Your
Five days after discharge, t he patient notices a petechial rash on her legs and 0 Answer
Folate deficiency
presents again for evaluation. Laboratory test ing reveals a hemoglobin
concentration of 11.4 g/dl (reference range, 12.0-16.0), a leukocyte count of Other
Thrombotic thrombocytopenic purpura
Answer
4200 per mm 3 ( 4500-11,000), a platelet count of 18,000 per mm 3
(150,000-350,000), a normal partial t hrom boplastin t ime and prothrombin
Other
time, normal fibrinogen levels, and normal renal functio n. Disseminated intravascular coagulation
Answer
Review of the peripheral-blood smear reveals marked reduct ion of the platelet
Other
count but normal platelet morphology. No schistocytes are present. Immune thrombocytopenia
Answer
In an inpatient who has received vancomycin and ceft riaxone, the most
likely cause of a new thrombocytopenia that develops during or shortly
after hospitalization is drug-induced thrombocytopenia.
DETAILED FEEDBACK:
CLOSE
Copynght © 2022 Massachusetts Medical Society_All rights reserved
G) You are in Demo mode. Results are not saved.
DETAILED FEEDBACK:
CLOSE
Copyright © 2022 Massachusetts Medical Society_ All nghts reserved
(D You are in Demo mode. Results are not saved.
A 28-year-old woman presents for evaluation of several episodes of new- Which one of the following management approaches is most appropriate
onset gingival bleeding. She was diagnosed with HIV infection 3 years ago for this patient?
and has adhered to her antiretroviral therapy wit h consistent virologic
suppression for several years. She also takes lisinopril to treat primary :You got it'
hypertension but uses no other medications. Her last CD4 count was 240
•
cells per mm 3 (reference range, 400-1600), and her HIV viral load is <20
Your
copies/ ml. High-dose glucocorticoid
Answer
Physical examination reveals no active gingival bleeding but petechiae limited
mainly to both lower extremities, with a few similar spots on her forearms, Other
Answer
Platelet transfusion
abdomen, and oropharynx. Her spleen is not palpable.
Laboratory testing shows the fol lowing results: Other
Answer
Splenectomy
Patient value Reference range
Other
Hemoglobin (g/dL) 12.7 12.0-16.0 Bone-marrow biopsy
Answer
Leukocyte count (per mm3) 4600 4500-1 1,000
Other
Rituximab
Platelet count (per mm 3) 12,000 150,000- 350,000 Answer
Her hepatic function, international normalized ratio, and partial- Immune thrombocytopenia (ITP) in people with HIV is initially treated with
thromboplastin time are all normal. A peripheral-blood smear shows a antiretroviral therapy; if ITP persists after virologic suppression other
significant decrease in mostly normal-appearing platelets, with a few t hat are treatments such as glucocorticoids are added.
enlarged.
DETAILED FEEDBACK:
CLOSE
Copynght © 2022 Massachusetts Medical Society. All nghts reserved
(D You are in Demo mode. Results are not saved.
A 41-year-old woman presents to t he emergency depart ment with abdominal In addition to administering flu ids and perform ing a t issue biopsy, what is
distention and pain. Her symptoms fi rst started 2 weeks ago and have the most appropriate next step in managing this patient?
worsened. She also reports unintended weight loss of 3 kg, ongoing chills, and
soaking night sweats. You got 1t1
She was born and raised in Mexico City and immigrated to the United States 3
years ago. She has no recollection of a tuberculosis diagnosis for herself or 60% Right Rasburicase
any close family members.
On physical examinat ion, she is afebrile and in moderate distress, with a heart 4% Other Trimethoprim-sulfamethoxazole
rate of 118 beats per m inute and a blood pressure of 116/73 mm Hg. Her
examinat ion is notable for several palpable cervical and inguinal lymph nodes, 4% Other Antitubercular therapy
as well as an enlarged spleen that is tender to palpat ion. Her abdomen is
mildly distended. 8% Other Bone-marrow biopsy
A rapid HIV test is positive. The patient is up to date with Covid-19
23% Other Antiretroviral therapy
vaccinations, and testing for SARS-CoV-2 is negative. Laboratory testing yields
the following results:
Hemoglobin (g/dL) 8.1 12.0-16.0 Spontaneous tumor lysis syndrome can occur in patients with Burkitt
Leukocyte count (per m m 3) 1300 4500-1 1,000 lymphoma and is treated wit h intravenous fluids and rasburicase.
CLOSE
Copynght © 2022 Massachusetts Medical Society_ All rights reserved
(D You are in Demo mode. Results are not saved.
CLOSE
Copynght © 2022 Massachusetts Medical Society. All rights reserved
(D You are in Demo mode. Results are not saved.
A 32-year-old woman presents for follow-up after a positive HIV test The test, Which one of the following treatments is most appropriate for this patient's
performed 2 weeks earlier at a healt h fair, was subsequently confirmed by a thrombocytopenia?
hospital laboratory. The patient has not yet initiated antiretroviral therapy.
On examinat ion, she is afebrile and appears well. There is no evidence of oral Not there yet..
thrush. Her spleen t ip is palpable 3 cm below the left costal margin in t he
midclavicular li ne. She denies bleeding. 74% Correct Ant iretroviral therapy
Laboratory testing shows a platelet count of 75,000 per mm 3
(reference range,
7 50,000-350,000) and a CD4 count of 350 cells per mm 3 (400-1600). Results 19% Wrong Prednisone 1 mg/kg daily for 2 weeks
from a complete blood count and blood chemistry test are otherwise normal.
A peripheral-blood smear shows reduced platelets with occasional large 1% Other Platelet transfusion
forms but no schistocytes.
2% Other Rituximab
4% Other Splenectomy
DETAILED FEEDBACK:
CLOSE
Copyright © 2022 Massachusetts Medical Society. All rights reserved
(D You are in Demo mode. Results are not saved.
A 32-year-old woman with sickle cell disease (Hb SS) presents for fol low-up. Which one of the following treatments would be most appropriate to
She is current ly asymptomatic but has had fou r acute pain episodes and reduce the frequency of this patient's acute pain episodes?
required three red-cel l t ransfusions during t he past year She has never had a
stroke or an episode of acute chest syndrome. Her only m edication is daily Not there yet...
folic acid, and she is up to date on all recommended vaccinations.
Correct
Answer
Hydroxyurea
Your
0 Answer
Prophylactic antibiotics
Other
Answer
Meperidine
Other
Answer
Iron chelation therapy
The first-line medication for reducing the frequency of acute pain episodes
in patients with sickle cell disease is hydroxyurea.
DETAILED FEEDBACK:
CLOSE
Copynght © 2022 Massachusetts Medical Society_ All nghts reserved
(D You are in Demo mode. Results are not saved.
An 18-year-old female college student with sickle cell anemia reports having Which one of the following treatments is indicated as first-line therapy to
one to two painful crises a year that require hospitalization. She does not reduce the risk of stroke in this patient?
recall having ever received a red-cell transfusion. She has intermittent pain in
her extremities and midback, which is relieved by nonsteroidal Not there yet..
anti inflammatory drugs or acetaminophen-hydrocodone.
The patient recalls an episode of confusion, slurred speech, and left-arm and 40% Correct Red-cell exchange transfusions
left-leg weakness that occurred in her early teens. Her symptoms resolved
within minutes, and she attributed it to not sleeping well the previous night. A 12% Wrong Aspirin
similar episode occurred again last year when she was studying for finals.
Again, she attributed it to sleep deprivation 1% Other Sumatriptan
Her only regular medication is norethindrone. She reports having occasional
migraine headaches that usually occur around her menstrual periods. She 45% Other Hydroxyurea
does not use tobacco or illicit drugs but does drink alcohol occasionally.
1% Other Iron chelation therapy
The most appropriate first-line treatment for reducing the risk of stroke in a
patient with sickle cell disease is prophylactic red-cell exchange
transfusions.
DETAILED FEEDBACK:
CLOSE
Copyright @ 2022 M assachusetts Medical Society. All nghts reserved.
G) You are in Demo mode. Results are not saved.
A previously healthy 20-year-old man reports a one-week history of easy Which one of the following next steps Is most appropriate for this patient?
bruising and bleeding from t he gums when flossing his teeth. He has no other
personal or family history of bleeding. He takes no medications and does not Not there yet. II
drink alcohol or use illicit drugs. He seems somewhat anxious during the
interview
61% Correct Initiate dexamethasone therapy
His vital signs are normal. Examination of the oropharynx reveals blood
around the gums and a few hemorrhagic bullous lesions on the mucosa. His 15% Wrong Test for platelet glycoprotein autoantibodies
skin shows numerous petechiae. There is no lymphadenopathy. The liver and
spleen are not palpable. 8% Other Refer for bone-marrow aspirat ion and biopsy
Laboratory testing reveals a platelet count of 10,000 per mm 3
(reference
range, 150,000-350,000), a hemoglobin level of 14 g/dL (7 3.5-17.5) with Other
Evaluate for the presence of ant iphospholipid
3%
normal indices, and a leukocyte count of 7500 per mm3 (4500-1 1,000) with antibodies
normal differential. A peripheral blood smear shows decreased platelets,
which are well granulated and larger than normal. Leukocyte and erythrocyte 13% Other Measure ADAMTS13 level
morphology are normal without any schistocytes. The serum lactate
dehydrogenase, prothrombin time, activated partial thromboplastin time, and
fibrinogen levels are normal. Hepatitis C virus and HIV tests are negative. KEY LEARNING POINT:
DETAILED FEEDBACK:
CLOSE
Copyright © 2022 Massachusetts Medical Society_ All nghts reserved
(D You are in Demo mode. Results are not saved.
A 22-year-old woman with sickle cell disease presents with chest and back In addition to supportive intensive care and ventilatory support, what
pain and progressive dyspnea. The back pain began 3 days ago and is management is most appropriate in this patient?
consistent with previous acute pain episodes. She notes no other inciting
illness or t rauma. Not there yet..
Her temperature is 38.6°C, her heart rate is 110 beats per minute, her blood
pressure is 139/88 mm Hg, and her respiratory rate is 30 breaths per minute. Correct
Answer
Perform an exchange transfusion of red cells
Her oxygen saturation increases from 88% while she is receiving 4 liters of
supplemental oxygen to 94% while she is wearing a nonrebreather face mask.
Your
She is hospitalized and started on intravenous fluids, opioid analgesics, and 0 Answer
Add vancomycin
intravenous ceftriaxone and azithromycin. However, she continues to have
chest discomfort and difficulty breathing and becomes progressively more Other
Answer
Administer intravenous immunoglobulin
dyspneic and hypoxic.
Chest radiography reveals right middle and right lower lobe consolidat ions Other
Answer
Administer intravenous dexamethasone
without any pleural effusions.
Other
Answer
I Administer intravenous heparin
Patients with sickle cell disease in whom moderate or severe acute chest
syndrome develops should receive short-term care consisting of red-cell
exchange transfusion.
DETAILED FEEDBACK:
CLOSE
Copyright @ 2022 Massachusetts Medical Society. All rights reserved.
(D You are in Demo mode. Results are not saved.
A 34-year-old woman is hospitalized for evaluation of acute jaundice. She was Which one of the following diagnoses is most likely In th is case?
prescribed nitrofurantoin for a urinary tract infect ion 5 days ago. Since then,
she has had progressive onset of dark urine, yellowing of her eyes, and pallor Not there yet . II
She reports a fami ly history of jaundice in her father
Her temperature is 38.4°C, her heart rate is 106 beats per m inute, and her Correct
Glucose-6- phosphatedehydrogenase deficiency
blood pressure is 113/89 mm Hg. She appears uncomfortable. There is no Answer
Direct bilirubin (mg/dL) 1.6 0.1-0.3 The most likely diagnosis in a patient treated wit h nitrof urantoin who
Haptoglobin (mg/dL) 5 16-199 subsequently develops findings consistent with a hemolytic anemia is
glucose-6-phosphate dehydrogenase deficiency.
Fibrinogen (mg/dL) 550 150-400
The pat ient's D-dimer test is normal, and her direct Coombs test is negative. DETAILED FEEDBACK:
The peripheral-blood smear shows multiple erythrocytes with bite-like The patient's presentation, peripheral-blood smear, high levels of lactate
deformities as well as small round inclusions without sickling. dehydrogenase and indirect bilirubin, and low level of haptoglobin suggest
hemolytic anem ia, which is the most common medical problem associated
with glucose-6-phosphate dehydrogenase (G6PD) deficiency. Hemolytic
anemia in patients w ith G6PD deficiency is often triggered by exposure to
medications, including dapsone, nitrofurantoin, phenazopyridine,
primaquine, and rasburicase. G6PD-deficient erythrocytes in combination
with high levels of oxidants cause a cross-linking of su lfhydryl groups on
globin chains; this causes their denaturation and the formation of Heinz
bodies (small round inclusions w ithin red cells that contain denatured
hemoglobin).
CLOSE
Copyright © 2022 Massachusetts Medical Society. All nghts reserved
(D You are in Demo mode. Results are not saved.
A 33-year-old woman wit h a history of hypothyroidism, hyperlipidemia, and Which one of the fo llowing next steps is most appropriate to address this
migraine headaches presents for a screeni ng physical. Her current patient's neutrophil count?
medicat ions are acetaminophen, atorvastatin, and levot hyroxine; she also
takes sumatriptan as needed. She feels well overall, other than m ild Not there yet...
arthralgias that are long-standing.
She identifies herself as Black on the patient intake questionnaire and reports 69% Correct Observe
that she is a kindergarten teacher. She has never smoked and rarely drinks
alcohol (<1 drink/week). Order flow cytometry to detect
8% Wrong
Her physical examination and vital signs are unremarkable. She has no glycosylphosphatidylinositol-anchored proteins
palpable adenopathy or organomegaly.
8% Other Order a bone-marrow biopsy
Routine laboratory testing reveals a leukocyte count of 1900 per mm3
(reference range, 4500-11,000) with an absolute neutrophil count of 1100 per
Measure serum vitamin B12, folic acid, and copper
mm 3 (1900-7600), a hemoglobin concentration of 13.1 g/dl (7 2.0-16 0), and 8% Other
levels
a platelet count of 334,000 per mm 3 (7 50,000-350,000). A peripheral smear
shows moderate neutropenia but no other cellular evidence of immaturity,
7% Other Discontinue sumatriptan
dysplasia, or blasts.
A review of the patient's electronic medical record reveal s that her absolute
neutrophil count has ranged from 800 to 1400 per mm 3 for the past 8 years. KEY LEARNING POINT:
She has had respiratory infections once or twice per year but no serious
infections requiring hospitalization. Asymptomatic isolated absolute neutrophil counts below the laboratory
reference range are characteristically seen in patients of certain ethn icities,
including people of African descent, who typically do not express the Duffy
allele (Fy(a-b-)).
DETAILED FEEDBACK:
CLOSE
Copyright© 2022 Massachusetts Medical Society. All rights reserved.
(D You are in Demo mode. Results are not saved.
A 30-year-old woman is urgently transferred from an outpatient endoscopy Which of t he following treatments is the most appropriate next step for t his
suite to the emergency department because of severe cyanosis, headache, patient after confirmation of the diagnosis?
tachycardia, tachypnea, and altered mental status. Her heart rate is 115 beats
per minute, and her blood pressure is 132/78 mm Hg. You got itl
When she presented to the endoscopy suite for an elective upper endoscopy, Your
0 Answer
Intravenous methylene blue
she felt well, and her oxygen saturation was 98%. She was premedicated with
an oropharyngeal spray of 20% benzocaine before t he procedure. Within Other
minutes, she became confused and was found to have an oxygen sat urat ion Answer
Exchange red-cell transfusion
of 78%. Administration of 100% oxygen failed to improve her cyanosis or pulse
oximetry reading. Other
Answer
Endotracheal intubation and ventilation
Other
Answer
Oral ascorbic acid
Other
Answer
I Hyperbaric oxygen
DETAILED FEEDBACK:
When the iron in hemoglobin is oxidized from ferrous (Fe2+) to ferric state
(Fe3+), the hemoglobin becomes methemoglobin. When all four heme
molecules are ferric instead of ferrous, the hemoglobin cannot bind oxygen.
However, clinically, usually only one or two of the iron atoms becomes ferric
under the influence of oxidizing drugs. In this situation, the affin ity of the
remaining ferrous-bearing heme molecules for oxygen increases, shifting
the hemoglobin-oxygen dissociation curve left and dim inishing oxygen
delivery to t he t issues. Exposure to certain oxidizing medications (e.g.,
benzocaine) can overwhelm t he cytochrome-b 5 reductase system and lead
to methemoglobinemia.
Two enzymatic processes reduce methemoglobin back to hemoglobin. The
CLOSE
Copyright © 2022 Massachusetts Medical Society_ All rights reserved
(D You are in Demo mode. Results are not saved.
A 20-year-old gravida 0, para 0 woman born in El Salvador presents with Which one of the following tests Is most appropriate In this patient?
fatigue. She has no known medical history and takes no medicat ions. She has
had regular menses since age 12 and average blood loss with each cycle. Her Not there yet... 1·
vital signs and physical examination are normal.
Laboratory results are as follows: 67% Correct Hemoglobin electrophoresis
Patient value Reference range
2% Wrong Vitamin B12 and folate levels
Leukocyte count (per mm 3) 7500 4500-11,000
Red-cell distribution width(%) 13.8 11.5-14.5 The diagnosis most suggested by microcytic anemia in a patient with a
normal red-cell distribution width and normal iron stores is either alpha-
Her peripheral-blood smear shows normal-appearing leukocytes and platelets thalassemia or beta-thalassemia.
w ith microcytic, hypochromic red cells and some target cells.
DETAILED FEEDBACK:
CLOSE
Copyright © 2022 Massachusetts Medical Society. All nghts reserved
(D You are in Demo mode. Results are not saved.
A 33-year-old woman presents to establish care. She reports several acute Which one of the following diagnoses 1s most likely 1n this patient?
episodes of pain during her lifetime, frequently involving her hips. She has
never required hospitalizat ion or intravenous opioids, and she has never been Not there yet. . II
pregnant. She has cousins with sickle cell disease who have had simi lar
painful episodes.
Correct
Hemoglobin SC disease
Physical examination shows an enlarged spleen that is palpable 3 cm below Answer
t he left costal margin.
Your
Laboratory testing shows a normal serum creatinine level and a hemoglobin 0 Answer
Sickle cell trait
level of 11.1 g/dl (reference range, 12.0-16.0). Her reticulocyte count is 4.5%
red cells (0 5-2 5) Other
Hemoglobin SS disease
Answer
A blood smear reveals many target cells and a few elongated sickle cells.
Other
Alpha-thalassemia minor
Answer
Other
Beta-thalassemia minor
Answer
The combination of mild anemia and infrequent pain crises in a patient wit h
splenomegaly suggests a diagnosis of hemoglobin SC disease.
DETAILED FEEDBACK:
Patients with hemoglobin (Hb) SC disease carry one gene for Hb S (the
sickle gene) and one gene for Hb C. The combination leads to a phenotype
that is less severe than hemoglobin SS disease but more severe than sickle
cell trait. Although the clinical presentations of sickle cell disease and sickle
cell trait can vary, hemoglobin electrophoresis can be performed to
distinguish between them. Patients with hemoglobin SC disease often have
splenomegaly, thought to be caused by the increased need of the spleen to
cull pitted and damaged erythrocytes.
Adults with even m ild sickle cell disease are unlikely to have splenomegaly
because they experience splenic infarcts during crises. The splenomegaly
is an important hint to the presence of SC disease. An additional clue is t he
presence of target cells.
CLOSE
Copynght © 2022 Massachusetts Medical Society. All nghts reserved
G) You are in Demo mode. Results are not saved.
A 35-year-old woman who is recovering from a hysterectomy for uterine What laboratory evaluation for heparin-induced thrombocytopenia (HIT), if
fibroids, complicated by wound cellulit is, is found on postoperative day 8 to any, is indicated for this patient at this time?
have a platelet count of 9000 per m m 3 (reference range, 150,000-350,000;
preoperative value, 200,000). The rest of her complete blood count is normal. Not there yet...
She has been receiving prophylactic dalteparin for 8 days and has no previous
history of exposure to heparin. In addition, she has been treated with
Correct
vancomycin for 5 days for a wound infection with methicillin-resistant No further testing is needed
Answer
Staphylococcus aureus. Her other current medications are omeprazole and
zolpidem. Your
0 Answer
Serotonin release assay
On physical examination, she has normal vital signs, including a heart rate of
76 beats per m inute and an oxygen saturat ion of 97% while she breathes Other
Heparin-induced platelet aggregation assay
ambient air Skin examination is notable for petechiae. Some oozing from the Answer
surgical site is evident. There is no leg swelling.
Other
Her peripheral blood smear shows no schistocytes or other findings t hat HIT enzyme-linked immunosorbent immunoassay
Answer
would suggest a microangiopathic hemolytic anemia. Her prothrombin time is
12.0 seconds (11.1-13.1 ), her partial thromboplastin time is 25.0 seconds Other
Answer
I ADAMTS13 activity level
(22.1 -35.1 ), and her serum lactate dehydrogenase level is normal.
The validated 4Ts scoring system helps to assess the pretest probability of
heparin-induced thrombocytopenia in a patient recently exposed to heparin.
DETAILED FEEDBACK:
CLOSE
Copyright© 2022 Massachusetts Medical Society. All nghts reserved.
G) You are in Demo mode. Results are not saved.
A 43-year-old woman with a history of hypertension, type 2 diabetes, and Which one of the following next steps is most appropriate in evaluating this
t ri geminal neuralgia presents for a routine physical examination. She reports patient's leukopenia?
moderate fatigue, difficulty sleeping, and knee pain (all as chronic issues). She
denies any recent infect ions. Her facial pain has improved substantially since Not there yet...
her neurologist started her on carbamazepine 6 weeks ago, but her energy
level remains poor. Her other medications are acetaminophen, metformin,
Correct Ongoing observation with serial hematologic
lisinopril, and hydrochlorothiazide. On average, she drinks one alcoholic
Answer surveillance
beverage 2 days per week.
Physical examination reveals no obvious abnormalities. Your Testing for HIV. hepatitis Band C viruses, vitamin B12 ,
Routine blood test ing is notable for a leukocyte count of 3200 per mm3
0 Answer folic acid, and copper
(reference range, 4500-7 7,000), with an absolute neut rophil count of 7400 per
mm 3 (7 900-7600) and an otherwise normal differential, a hemoglobin level of Other
Answer
Bone-marrow aspiration and biopsy
7 6.0 g/dl (7 2.0-7 6.0), and a platelet count of 327,000 per mm 3
(7 50,000-350,000). Pathology review of her peripheral smear shows no
Other Sequential discontinuation of medications, beginning
obvious abnormalities. A review of her electronic medical record reveals that Answer with carbamazepine, and monitoring for improvement
her leukocyte counts during the past 70 years have generally ranged from
5000 to 9000 per mm 3. Other
Answer
Flow cytometry of the blood
DETAILED FEEDBACK:
CLOSE
Copyright © 2022 Massachusetts Medical Society_All nghts reserved
(D You are in Demo mode. Results are not saved.
A 53-year-old man presents for a routine checkup. He feels fa irly well and has Which one of the following diagnoses 1s most likely 1n this case?
not had any recent infections. He has a long history of seropositive
rheumatoid arthritis with erosive disease. He previously required treatment Not there yet .. JI
with prednisone and methotrexate but has been off treatment for a year.
On physical examination, the patient's spleen is palpable 3 cm below the left 84% Correct Felty syndrome
costal margin, but the abdominal examination is otherwise normal. There is
no lymphadenopat hy and no evidence of active synovitis. 77% Wrong Myelodysplastic syndrome
Laboratory testing yields the fo llowing results:
7% Other Portal vein thrombosis
Patient value Reference range
I Other
Hemoglobin (g/dL) 72.5 73.5-77.5 3% Diffuse large-B-cel l lymphoma
CLOSE
Copyright © 2022 Massachusetts Medical Society. All nghts reseived.
(D You are in Demo mode. Results are not saved.
A 28-year-old man returns for follow-up after treatment for deep-vein Which one of the followi ng diagnoses Is most likely In this case?
thrombosis. The deep-vein thrombosis was complicated by a nonocclusive
pulmonary embolus in the segmental branches of the left lower lobe and Not there yet. II
lingula, as seen on a CT angiogram of his thorax. He has no history of recent
trauma or long-distance travel, and he does not smoke or use illicit drugs.
67% Correct I Beta-thalassemia intermedia
The patient was t reated with therapeutic low-molecular-weight heparin and
transitioned to warfarin 5 mg daily by mouth. (Rivaroxaban was initially 6% Wrong I Von Willebrand disease
recommended, but there were cost concerns.) His international normalized
ratio is 2.3, within the target range of 2.0 to 3.0. 9% Other Antiphospholipid syndrome
Laboratory testing yields the following results
22% Other Factor V Leiden mutation
Patient value Reference range
Leukocyte count (per mm3) 10,600 4500-1 1,000 3% Other Factor VII I inhibitor
Mean corpuscular volume (µm 3) 66 80-100 Patients with beta-thalassemia intermedia are at increased risk for
Red-cell distribution width(%) 12.8 11.5-14.5 developing venous thromboembolism.
CLOSE
Copyright © 2022 Massachusetts Medical Society_All rights reserved
(D You are in Demo mode. Results are not saved.
medications. His HIV viral load recently increased to more than 50 copies/ml,
Other
and his CD4 count recently decreased to <100 cells per mm3 (reference range, HIV infection
Answer
400-1600).
On day 2 of his hospitalization, he is diagnosed with cytomegalovirus enteritis, Other
Answer
Mycobacterium avium complex infection
according to a biopsy of mucosal ulcers during a colonoscopy. On hospital
day 3, he is started on intravenous ganciclovir.
Other
Treatment with efavirenz, tenofovir, and emtricitabine
By day 10 of his hospitalization, his diarrhea has improved significantly and he Answer
is afebrile.
Other
Laboratory testing shows t he following results: Cytomegalovirus infection
Answer
- -
At admission At day 10 Reference range
Leukocyte count (per mm3) 4400 1400 4500- 11,000 A serious hematologic adverse effect of treatment with ganciclovir is
Absolute neutrophil count (per 2500 800 1900-7600 neutropenia.
mm 3)
DETAILED FEEDBACK:
Platelet count (per mm 3) 110 X 10 3 103 X 103 1 50-400 X 103
Total bilirubin (mg/dl) 0.5 0.6 0.3-1.0 Adverse medication effects are t he most common cause of neutropenia in
patients with HIV infection and can usual ly be identified when the onset of
Creatinine (mg/dL) 1.3 1.2 0.8-1.3
neutropenia occurs after initiation of a new medication. Because
ganciclovir frequently causes neutropenia, monitoring of complete blood
counts during ganciclovir therapy is recommended.
Both HIV infection and antiretroviral therapy can cause neutropenia,
especially if zidovudine is used. However, t his is unlikely when the onset of
neutropenia is acute.
Cytomegalovirus infection can cause neutropenia as a result of direct bone-
marrow involvement, but cytomegalovirus-associated neutropen ia should
improve, not deteriorate, following appropriate treatment.
MycobActerium ivium comolex infection is unlikely in a oatient who is
CLOSE
Copyright © 2022 Massachusetts Medical Society_ All rrghts reserved